You are on page 1of 113

www.gradeup.

co

1. Principle:

The owner of immovable property is entitled to the column of airspace above the surface. However, the owner's right
to air and space above his land is restricted to such height as is necessary for the ordinary use and enjoyment of his
land and the structures on it.

Facts:

Galaxy Cable TV Network Company is providing cable connections to their customers. One of the cables passes
over the house of Mr. Vasanth Bhat. He is not a customer of the Network Company. The cable is neither attached to
his house nor to any projection thereof. It is at a distance of 20 feet above the terrace of Mr. Bhat's two storied house.
Because of the cable, Mr. Bhat's son Sachin is unable to fly a kite from the terrace. Mr. Bhat requested the Network
Company to change the position of the cable. But the company did not bother to change it. One evening Mr. Bhat cut
the cable and cleared the airspace above his house. The Network Company suffered a loss of about 1000/-. They
bring a legal action against Mr. Bhat for recovery of loss suffered.
B. The Network Company will not succeed because Mr.
A. The Network Company will succeed because the cable
Bhat has every right to ensure proper enjoyment of his
  was not interfering with the ordinary use and enjoyment
property by removing objects causing trespass in the air
of Mr. Bhat's property.
above his property to a reasonable extent.
C. The Network Company will succeed because laying
  cables is widely practised in all cities like electricity 'and D. None of the above.
telephone wires.
2. Principle:

Everybody is under a legal obligation to take reasonable care to avoid act or omission which he can foresee would
injure his neighbour. The neighbour for this purpose is any person whom he should have in his mind as likely to be
affected by his act.

Facts:

Ram, while rushing to board a moving train, pushed Shyam who was walking along with a heavy package,
containing fire crackers. As a result, the package slipped from his hand and the crackers exploded injuring a boy-
standing closely. A suit was filed against Ram, by the boy, claiming damages.
  A. Ram is not liable, because he did not know anything B. Ram is not liable, because Shyam should not have
www.gradeup.co

about the contents of the package. carried such a package in a crowded place like Railway
station.
C. Ram is liable, because Ram is under an obligation not
  D. None of the above.
to push Shyam.
3. Principle:

An occupier is not, normally liable to a trespasser except in respect of willful act intended to cause harm or done with
reckless disregard.

Facts:

Kishan lal was running a dairy farm in his house. A part of his farm was used by the people as a short cut to get into
the nearby railways station. Kishanlal never liked it and put up a board that "All trespassers will be prosecuted". But
he actually tolerated them, because quite a few of them patronized his business. One day, a person, who was crossing
the farm to get into the railway station, was attacked by a bull belonging to the farm. The injured person tiled a suit
against Kishanlal.
A. Kishanlal is not liable in view of the clear notice B. Kishanlal is liable, because he in fact allowed the
 
against trespassers. people to use his premises.
C. Kishanlal is not liable to the people other than his
  D. None of the above
customers.
4. Principle:

No person shall be convicted of any offence except for violation of a law in force at the time of the commission of
the Act charged as an offence, nor be subjected to a penalty greater than that which might have been inflicted under
the law in force at the time of the commission of the offence.

Facts:

A boy of 16 years of age was convicted for committing an offence of house trespass and outraging the modesty of a
girl aged 7 years. The magistrate sentenced him for 6 months rigorous imprisonment and also imposed fine. After
Judgment, the Probation of Offenders Act, 1958 came into force. It provided that a person below 21 Years of age
should not ordinary be sentenced to imprisonment. Now the boy claims the benefit of the Probation of Offenders Act.
A. He should not get the benefit of Probation of Offenders B. The rule of beneficial interpretation required that ex
 
Act. post facto law could be applied to reduce the punishment.
  C. A boy below the age of 21 years is treated as minor and D. None of the above.
www.gradeup.co

so he should not be punished.


5. Principle:

Section 497 of IPC states that 'Whoever has sexual intercourse with a person who is and whom he knows or has
reason to believe to be wife of another man, without the consent or connivance of that man, such sexual intercourse
not amounting to the offence of rape, is guilty of the offence of adultery, and shall be punished with imprisonment of
either description for a tern which may extend to five years, or with fine, or with both. In such case the wife shall not
be punishable as an abettor'.

Facts:

Mohan was invited by Revary, wife of Shyam to have sexual intercourse with her. Knowing about it Shyam filed a
case against Mohan and Revati.
B. Revati cannot be punished as an abettor but she can be
  A. Mohan and Revati are liable for the same offence.
punished for the main offence.
  C. Revati is not liable to be punished for any offence. D. None of the above.
6. Principle:

When an act is abetted and a different act is done, the abettor is liable for the act done, in the same manner and to the
same extent as if he had directly abetted it.

Facts:

A instigates a child to put poison into the food of Z, and gives him poison for that purpose. The child, in consequence
of the instigation, by mistake puts the poison into the food of Y, which is by the side of that of Z.
  A. A is liable for abetting to kill Z. B. A is not liable.
  C. A is liable for abetting to kill Z and for murder of Y. D. None of die above.
7. Principle:

A person is liable for Murder when he does any act with such intention or knowledge and under such circumstances
that, if he by that act caused death, he would be guilty of murder.

Facts:

Motilal with the intention to kill Ramlal gave opium to Ramlal in such a quantity as in normal case sufficient to
cause death. Ramlal was an opium addict who enjoyed it and survived.
www.gradeup.co

  A. Motilal is liable for attempt to murder. B. Motilal is not liable for attempt to murder.
  C. Motilal is liable for abetment to suicide. D. None of the above.
8. Principle:

A child below 7 years is regarded as doli incapax which means a person incapable of having wrongful intention.

Facts:

Rohit, aged 6 years caused hurt on the head of Diana, his neighbour, who sustained serious injuries.
  A. Rohit is liable to be punished for hurt. B. Rohit is not liable to punished for hurt.
C. Rohit is liable only if it is proved that he is mature
  D. None of the above.
enough to understand the consequences of his acts.
9. Principle:

Law does not take account of trifles.

Facts:

A has an inkpot with him. B wants to fill his pen from that inkpot. A does not allow, still B takes ink from it.
  A. A is liable to be punished. B. A is not liable to punished.
  C. A is liable for damages. D. None of the above.
10. Principle:

Ignorance of fact is an excuse but ignorance of law is not an excuse.

Facts:

N, a nurse in a hospital gives P, a patient, poison thinking it to be a medicine which killed the patient
  A. N is liable for murder. B. N is not liable for murder.
C. N is liable for culpable homicide for not taking
  D. None of the above.
reasonable care.
11. Principle:

If any person contaminated any source of public water in such a way that it becomes non-consumable, he shall be
punished.
www.gradeup.co

Facts:

Raghuram owns a chemical factory in Delhi._Ignorance of fact is an excuse but ignorance of law is not an excuse.
FACTS_ N, a nurse in a hospital gives P, a patient, poison thinking it to be a medicine which killed the patient
  A. N is liable for murder. B. N is not liable for murder.
C. N is liable for culpable homicide for not taking
  D. None of the above
reasonable care.
12. Principle:

Act of a person incapable of judgment by reason of intoxication caused against his will is excused.

Facts:

A night watchman, during his duty took a minor girl and in his attempt to rape placed his hands upon her mouth and
she died due to suffocation. He took a defence that he was unable to understand the consequences as he was heavily
drunk. In this case
A. Accused is guilty of offence since he was voluntarily B. Accused is not guilty of offence since he was in
 
drunk. capable of Judgement.
  C. Accused is guilty of attempt to Rape only. D. None of the above.
13. Principle:

If both the parties agree upon the same thing in the same sense, the parties are bound by their agreement.

FACTS :

Sanjay wrote to Hrithik offering to sell his horse for. 20,000/-. Hrithik wrote back, "I agree to purchase your black
horse for 20100".
A. The parties are bound by their agreement as they agree B. The parties are not bound by the agreement as the
 
on the price and also on the goods for sale. object is uncertain.
C. The parties are bound by their agreement as the colour
  D. None of the above.
of the horse is only a question of detail.
14. Principle:

The Supreme Court shall, to the exclusion of any other court, have original jurisdiction in any dispute—(a) between
the Government of India and one or more States; or (b) between the Government of India and any State or States on
www.gradeup.co

one side and one or more other States on the other; or (c) between two or more States, if and in so far as the dispute
involves any question (whether of law or fact) on which the existence extent of a legal right depends:

Facts:

State of Karnataka filed a suit against Unit of India before Sub Court, Mysore for recovery ( an amount of 18,686
with interest and cost damages for delayed delivery of some articles Indian Railway.
A. Suit is not maintainable since it is a dispute between
  B. Suit is maintainable since Railway is not 'States
union and state
C. Suit is maintainable since it is dispute arising out of a
  D. None of the above.
contract between a carrier.
15. Principle:

To be held guilty of an offence, one should have done the act that causes the inter result.

Facts:

A with, the intention; to murder B stab him repeatedly with knife. B is taken to hospital and is found out of danger.
Thereafter, due to negligence of the doctor, B's wounds are infected and he requires surgical intervention. During tl
time of operation requiring to remove his iniuries infected leg, B died on account of administration.
B. A is not guilty of murder of B but may be guilty of
  A. A is guilty of murder of B
attempt to murder.
C. A is not guilty of murder of B buc t-tu. De gull of
  D. None of the above.
causing hurt.
16. Principle:

Theft is the dishonest moving of property with the intention of taking it out for the person's possession without his
consent.

Facts:

A gives his woollen coat to a dry cleaner along with his wife's sarees for the purpose of dry cleaning. He is told to
collect the clothes after two days. When he comes after two days, he finds that he does not have enough money to
pay to the dry cleaner. But since due to the winter, he needs the coat desperately, he surreptitiously places the coat
near his other goods so that he can quietly take it Without the knowledge of the dry cleaner
www.gradeup.co

  A. A is guilty of theft. B. A is not guilty of theft.


C. A is not guilty of theft but has to pay compensation to
  D. None of the above.
the dry cleaner.
17. Principle:

Time is of essence of the contract.

Facts:

A printing press, agrees to print-out question papers for an examination, the date of which has already been
announced. The printing is delayed first for the break down of the machinery and thereafter due to lack of power
supply. Tilt! University insists that the work must be completed within the time omit. Decide.
A. There was no specific stipulation in the agreement that B. The non-completion of the job was due to
 
time was of the essence of contract. circumstances beyond control of owners of the press.
C. It was clear from the nature of the job that time was of
  D. None of the above.
the essence of the contract.
18. Principle:

A Contract is an agreement enforceable by law. All agreements are contracts on the off chance that they are made
with free consent by parties able to go into an agreement for a legal consent and with a legitimate item.

Facts:

Rajat offered to purchase Shyaam's Car for Rs. 5 lakhs yet Shyaam can't. In this manner Rajat took steps to slaughter
Shyaam and at last he consented to the deal. Shyaam in this manner repealed from the agreement. Rajat suit to
uphold the agreement—Decide
A. Will succeed on the grounds that Rajat was offering B. Will succeed in light of the fact that purchasing and
 
legal thought for the vehicle selling of vehicle is legitimate
C. Will succeed in light of the fact that the two players D. Will come up short on the grounds that Rajat had to
 
have ability to contract consent to the agreement.
19. Principle:

A Contract is an agreement enforceable by law. All agreements are contracts in the event that they are made with free
assent by parties equipped to contract for a legitimate consideration and with a legitimate object.
www.gradeup.co

Facts:

Jayant, a little youngster of 27 years with no consideration consents to give Julie Rs. 10, 000/ - . Jayant neglects to
satisfy his guarantee. Julie sues Jayant for the sum. Choose
A. Juliy will prevail as Jayant caused the guarantee B. Ms. Jooly will come up short as the agreement is
 
willingly. without Consideration.
C. Ms. Jooly will prevail as Jayant has the ability to make D. Ms. Jooly will prevail as the cash isn't being paid for
 
the agreement. any unlawful item.
20. Principle:

Indian Constitution enables the President to appoint the Judges of the Supreme Court of India and High Courts.

Fact:

There is a tussle between the Council of Minister which additionally incorporates Prime Minister and President.
Members selected Mr. Adarshnath as the Justice of Supreme Court of India and Mr. Gargik designated as Judge of
Delhi High Court by the Prime Minister of India. Choose
A. Appointment of Mr. Gargik is legitimate according to B. Appointment of Mr. Adarshnath is substantial
 
law. according to the Constitution of India.
C. Appointment of Mr. Adarshnath and Mr. Gargik void
  D. None of the above mentioned.
ab initio.
21. Principle:

All residents will reserve the privilege to the right to speak freely of discourse and articulation under Article 19 of the
Constitution of India and it is a Fundamental Right.

Fact:

Sanjay, a popular english essayist and speaker reprimanded another author Ramesh: "The epic of Ramesh is
unreasonable and profane, his brain is polluted, he is a free character, he ought to compose fair and great novel."
Decide Can Ramesh be sued for maligning?
A. He isn't at risk since he has quite recently B. He is at risk to be sued for maligning if his assertion,
 
communicated his own perspectives was false or said in mala fide aim
C. He can't be held subject since he has crucial right to the D. He can't be sued, in light of the fact that both are author
 
right to speak freely of discourse and articulation and writers and both can condemn one another.
www.gradeup.co

22. Principle:

Each Indian citizen has a fundamental right to continue any exchange or business or calling of his decision subject to
the inconvenience of sensible limitations by the State.

Fact:

As per a request passed by the Institute of Company Secretary of India (ICSI), preclude an individual from Practice
the calling of Company Secretary except if such individual passed the Company Secretaryship Course and hold a
testament of training prom the ICSI. Mahesh, a Trainer Program passed understudy of Company Secretaryship
Course difficulties the boycott as it disregards his entitlement to convey his calling. Choose
B. The boycott is defended, as the right of Mahesh to
  A. ICSI has a better right than force boycott.
convey his calling isn't absolute.
C. The boycott isn't legitimized, as the Institute of
  Company Secretary of India (ICSI) can't deny any D. None of the above mentioned.
individual of his entitlement to carry on his calling.
23. Principle:

A Contract is an agreement enforceable by law. All agreements are contracts on the off chance that they are made
with free consent by parties able to go into an agreement for a legal consent and with a legitimate item.

Facts:

Rajat offered to purchase Shyaam's Car for Rs. 5 lakhs yet Shyaam can't. In this manner Rajat took steps to slaughter
Shyaam and at last he consented to the deal. Shyaam in this manner repealed from the agreement. Rajat suit to
uphold the agreement—Decide
A. Will succeed on the grounds that Rajat was offering B. Will succeed in light of the fact that purchasing and
 
legal thought for the vehicle selling of vehicle is legitimate
C. Will succeed in light of the fact that the two players D. Will come up short on the grounds that Rajat had to
 
have ability to contract consent to the agreement.
24. Principle:

A Contract is an agreement enforceable by law. All agreements are contracts in the event that they are made with free
assent by parties equipped to contract for a legitimate consideration and with a legitimate object.
www.gradeup.co

Facts:

Jayant, a little youngster of 27 years with no consideration consents to give Julie Rs. 10, 000/ - . Jayant neglects to
satisfy his guarantee. Julie sues Jayant for the sum. Choose
A. Juliy will prevail as Jayant caused the guarantee B. Ms. Jooly will come up short as the agreement is
 
willingly. without Consideration.
C. Ms. Jooly will prevail as Jayant has the ability to make D. Ms. Jooly will prevail as the cash isn't being paid for
 
the agreement. any unlawful item.
25. Principle:

Indian Constitution enables the President to appoint the Judges of the Supreme Court of India and High Courts.

Fact:

There is a tussle between the Council of Minister which additionally incorporates Prime Minister and President.
Members selected Mr. Adarshnath as the Justice of Supreme Court of India and Mr. Gargik designated as Judge of
Delhi High Court by the Prime Minister of India. Choose
A. Appointment of Mr. Gargik is legitimate according to B. Appointment of Mr. Adarshnath is substantial
 
law. according to the Constitution of India.
C. Appointment of Mr. Adarshnath and Mr. Gargik void
  D. None of the above mentioned.
ab initio.
26. Principle:

An infringement of a legal right, with or without injury, frames the beginning of the offense of tort.

Fact:

Mr. Ketan sets up an instructing class for Company Secretary Students of Executive Program and charges Rs.10,
000/ - every year as educational expenses. Mr. Ketan's neighbor Mr. Kalia builds up another instructing class for
Company Secretary Students of Executive Program along these lines making an opposition between them. This
powers Mr. Ketan to diminish his educational expenses to Rs. 7000/ - every year. Choose Can Mr. Ketan guarantee
harms from Mr. Kalia for the misfortune caused to him?
A. Yes, he can as Mr. Kalia has disregarded his Legal
  B. No, Mr. Ketan has diminished the charges all alone.
Right
  C. No, on the grounds that however, there was damage D. None of the above mentioned.
www.gradeup.co

there was no lawful injury


27. Principle:

Company implies an organization joined under the Companies Act, 2013, or under any past organization law.

Facts:

(i) ABC Limited is joined under the Companies Act, 1956; (ii) ABC LLP consolidated under Limited Liability
Partnership Act, 2008;(iii) ABC and Co enrolled under Partnership Act, 1832 and (iv) ABC Charitable Trust set up
under Trust Act, 1882. Choose Which of the above element as referenced in verifiable circumstance is an
organization?
  A. ABC Limited B. ABC LLP
  C. ABC and Co. D. ABC Charitable Trust.
28. Principle:

A master will be liable for the unjust demonstrations of his servants throughout his business.

Facts:

ABC Limited is a Non-Banking Financial Company (NBFC) is enrolled with Reserve Bank of India to acknowledge
store from public. Mr. Shyam delegated as approved specialist by ABC Limited to gather store cash from a few
people on consistent schedule. Mr. Shyam, gathering stores from individuals on regular routine. One day he
vanished. One Ms. Shyamlee , who had been giving over her store cash to Mr. Shyam found that almost for a month
before his vanishing, he was not keeping her cash by any means. Ms. Shyamlee when moved toward the ABC
Limited, the NBFC stood up that Mr. Shyam was not its representative, he is his representative and subsequently,
ABC Limited isn't liable for his unfortunate behavior. Ms. Shyamlee records a suit against the ABC Limited.
Conclude Who is at risk?
  A. ABC Limited B. Mr. Shyam
  C. Ms. Shyamlee D. None of the above mentioned.
29. Principle:

The member from both Lok Sabha and Rajya Sabha are qualified to be Ministers of the Union Government.

Fact:
www.gradeup.co

Mr. Smash Sing is siting individual from Upper House of Parliament and as Recommendations of Prime Minister of
India, President of India designated Mr. Smash Singh as Minister for the Ministry of Education. Choose
Appointment of Mr. Slam Singh as Minister for the Ministry of Education is legitimate?
  A. Yes B. No
  C. President of India can't be delegated D. Prime Minister of India can't be suggested
30. Principle:

Tort is a common wrong for which the cure is a fact-based law activity for unliquidated damages and which isn't
solely the set of an agreement or the break of a trust or other simply fair commitment.

Fact:

Mr. Sunil has six pet dogs which are savage and bark a ton. Mr. Surendra is his nearby neighbor and feels that he
can't make the most of his property because of the presence of an enormous number of dogs in the adjoining house.
He is mulling over starting a legal proceeding against Mr. Sunil. Mr. Surendra can: Decide
A. Initiate proceeding for violation of agreement against B. Initiate proceeding for unliquidated damages under
  Mr. Sunil as there is a suggested contract between the misdeeds in light of the fact that there is no agreement
neighbors. between the neighbors.
C. Initiate criminal activity against Mr. Sunil in light of D. Not be fruitful as Mr. Sunil has an unlimited option to
  the fact that having an enormous number of fierce dogs is make the most of his own property and has no obligation
a criminal demonstration. towards Mr. Surendra.
31. Principle:

When two or more persons make a joint promise the promisors may in the absence of express agreement to the
contrary compel any one or more of such joint from users to perform the whole of the promise. Each promisor may
compel contribution. Each of two or more joint promises Mein compel every other joint promises to contribute
equally with himself to the performance of the promise and less a contrary intention appears from the contract. When
two or more persons have made a joint promise then unless a contrary intention appears by the contract all such
persons during their joint lives and after the death of any of them his representatives jointly with the survivor and
after the death of the last survivor the representative of all jointly must fulfill the promise. If any one of two or more
joint promises makes default in such contribution the remaining joint promises must bear the loss arising from such
default in equal shares. Where two or more persons have made a joint promise release of one of such joint promisor
by the promisee does not discharged the other joint promisor neither does it free the joint promises so released from
responsibility to the other joint promisor or joint promisees.
www.gradeup.co

Facts:

Amitabh, Bhupendra, and Chintan are friends who borrowed Rs 12000 from Devesh for party out late at night. They
promised to pay to him the amount within reasonable time. Finally they returned and Chintan was found sufficiently
sick. Devesh seeks to recover the amount from the three.

Devesh has some enmity with Bhupendra. To take his case he seeks to recover the whole of it from Bhupendra.
Choose the correct.
A. He may recover the whole of the amount from B. He may not recover whole of the amount from
 
Bhupendra. Bhupendra as there are two more promisors here.
C. The practice contemplated of Devesh recovering whole
  of it from Bhupendra shall be against the basic principles D. Question does not provide sufficient data.
of Justice.
32. Principle:

When two or more persons make a joint promise the promisor may in the absence of express agreement to the
contrary compel any one or more of such joint from users to perform the whole of the promise. Each promisor may
compel contribution. Each of two or more joint promises Mein compel every other joint promises to contribute
equally with himself to the performance of the promise and less a contrary intention appears from the contract. When
two or more persons have made a joint promise then unless a contrary intention appears by the contract all such
persons during their joint lives and after the death of any of them his representatives jointly with the survivor and
after the death of the last survivor the representative of all jointly must fulfill the promise. If any one of two or more
joint promises makes default in such contribution the remaining joint promises must bear the loss arising from such
default in equal shares. Where two or more persons have made a joint promisors release of one of such joint
promisors by the promisors does not discharged the other joint promisor or joint promises neither does it free the
joint promises so released from responsibility to the other joint promasior or joint promisors.

Facts:

Amitabh, Bhupendra and Chintan are friends who borrowed Rs 12000 from Devesh for party out late at night. They
promised to pay to him the amount within reasonable time. Finally they returned and Chintan was found sufficiently
sick. Devesh seeks to recover the amount from the three.

Imagine that Bhupendra is not present in the meeting where in it is decided that there are three joint promisors. Hi
event does not attend the party and is no more a good friend of Amitabh and Chintan. In addition to the same he
www.gradeup.co

announces that he isn't borrowing any money. Choose the correct option.
B. He may not be compelled to pay his share as his
  A. His representative may be approached for the amount.
intention is different on the face of the contract.
C. He may be compelled by the two promises to pay his
  D. All of the above
share to Devesh
33. Principle:

When two or more persons make a joint promise the promisee Mein in the absence of Express agreement to the
contrary compel any one or more of such joint from users to perform the whole of the promise. Each promiser may
compel contribution. Each of two or more joint promises Mein compel every other joint promises to contribute
equally with himself to the performance of the promise and less a contrary intention appears from the contract. When
two or more persons have made a joint promise then unless a contrary intention appears by the contract all such
persons during their joint lives and after the death of any of them his representatives jointly with the survivor and
after the death of the last survivor the representative of all jointly must fulfill the promise. If any one of two or more
joint promises makes default in such contribution the remaining joint promises must bear the loss arising from such
default in equal shares. Where two or more persons have made a joint promise release of one of such joint promises
by the promisee does not discharged The Other joint promise array or joint promises neither does it free the joint
promises so released from responsibility to the other joint promasidor joint promisers.

Facts:

Amitabh, Bhupendra, and Chintan are friends who borrowed Rs 12000 from Devesh for party out late at night. They
promised to pay to him the amount within reasonable time. Finally they returned and Chintan was found sufficiently
sick. Devesh seeks to recover the amount from the three.

Imagine that Chintan died after being sick. What shall in ordinary circumstances be the distribution of amount to be
paid to Devesh?
B. Amitabh and Bhupendra must pay Rs. 4000 and should
A. Amitabh and Bhupendra must pay equally. No
  company in existing representative of Chintan to pay for
representative may be involved here.
him.
C. Amitabh and Bhupendra must pay only their amounts
  and shouldn't be bothered about amount that was to be D. All of the above
paid by Chintan.
34. Principle:
www.gradeup.co

Promises which form the consideration or part thereof for Each Other under section 2(f) are called Reciprocal
promises.

Facts:

Dev promises to give diamond ring to Anamika and in return, Anamika, who was a professional singer, promises him
to sing for him for two consecutive days. What kind of promise/consideration is this?
  A. Voidable B. Void
  C. Reciprocal promise D. Wagering Contract
35. Principle:

A condition to a contract can also be complied with after happening the event to which such condition is attached.

Facts:

Parul promises to pay Rs. 5000/- to Queen on the condition that he shall marry with the consent of R, S, and T. Q
marriage without the consent of R, S, T but obtained their consent after the marriage.
  A. The condition is illegal. B. Q have fulfilled the condition.
  C. Q has not fulfilled the condition. D. Q's marriage is not valid.
36. Principle:

Wagering agreements are void. Collateral agreements to wagering contract are valid.

Facts:

XYZ Bank lands Rs 4000/- to Sabu in order to enable him to award as prize to Randeep who is the winner of horse
race. Later Sabu refuses to pay the price is stating that horse racing is wagering agreement. Can XYZ Bank recover
money?
A. Yes as it is only a Collateral agreement to horseracing
  B. No as it is a wagering contract.
and therefore the bank can recover the money from Sabu.
C. Horse racing is illegal and therefore xyz that Bank D. Bank can recover money from Sabu so that payment of
 
cannot recover anything from Sabu. prize money can be made to Randeep.
37. Principle:

Law never enforces an impossible promise.


www.gradeup.co

Facts:

T made a promise to Q to discover Treasure by magic.


  A. Law will enforce the promise only at the option of Q B. Law will not enforce the promise
  C. Law will enforce the promise D. Law will enforce the promise only at the option of P
38. Principle:

A bailment is the delivery of goods by one person to another for some purpose upon a contract that they shall when
the purpose is a accomplished be returned or otherwise disposed of according to the direction of the person
delivering them.

Facts:

A lot of jewelry and valuable of Ram was found in house which were seized by police as income tax authorities
storm his house and found and tagged valuable. The valuable were to remain with the police to proper order of the
court in this matter. The valuable and jewelry were bailed by Ram to the police.
  A. The contract is not a bailment contract. B. The contract is a bailment contract.
  C. Void at the option of Ram. D. None of the above.
39. Principle:

Any contract made for an unlawful consideration is void in law.

Facts:

Ganga enters into a contract with Nikki to murder his wife, Priya for a sum of Rs. 5 lakhs. Ganga agrees to pay the
amount to Nikki in return of the murder. Is it a valid contract?
A. The contract is voidable at the option of Nikki due to
  B. The contract is void due to unlawful consideration.
the risk involved in it.
C. The contract is valid and Nikki must perform the act of D. The contract is uncertain as the manner of murdering
 
murdering Priya. Priya is not specified in it.
40. Principle:

With the counter proposal made by the party, the original offer ceases to exist.

Facts:
www.gradeup.co

Badal offered to sell his car to Pranav at the price of Rs. 3 lakhs. Pranav replies that he is ready to buy the car at Rs.
2.5 lakhs only. Badal refuses to sell at that price. Later, Pranav goes to Badal with Rs. 3 lakhs to buy the car. Is the
offer to Rs. 3 lakhs still valid?
A. The offer of Rs. 3 lakhs is not valid with a counter B. The offer of Rs. 3 lakhs still stands for Pranav to buy
 
offer of Pranav being made. the car.
C. The offer stands as it was made by owner of the car i.e.
  D. The offer can be renegotiated by the buyer and seller.
Badal himself.
41. The Union Government has increased the Minimum Support Price (MSP) by Rs 50 per quintal for which of the
following crops?
  A. Rabi crops. B. Kharif crops.
  C. Zaid crops. D. Cash crops.
42. The key features of “Pradhan Mantri Ghar Tak Fibre Scheme” launched by the Union Government are:

(I) Under this scheme, all the villages of Bihar will be connected through Optical Fibre Internet Service. For this, the
Common Service Centers (CSC) under the Ministry of Electronics & Information Technology (MeitY) has
undertaken the task of laying optical fibre networks.

(II) The Project would entail implementation of 1 Wi-Fi and 5 free of cost connections to Government institutions
like Primary Schools, Anganwadi Centers, Aasha workers, Jeevika Didi, etc.

(III) This Project will lead digital services like e-Education, e-Agriculture, Tele-Medicine, Tele-law and other social
security schemes to be easily available to all citizens of Bihar.

(IV) The CSC is taking the initiative through its village level Entrepreneurs (VLEs) under the BharatNet Project.
  A. I, II B. I, III, IV
  C. I, IV D. I, II, III, IV
43. The name of the high-altitude tunnel which is built by the National Highways and Infrastructure Development
Corporation Limited (NHIDCL), which is going to connect the Union Territory of Ladakh and Lahaul and Spiti
district of Himachal Pradesh
  A. Atal Tunnel. B. Barog Tunnel.
  C. Shinku La Tunnel. D. Zoji-la Tunnel.
44. Which is the world’s longest serving Aircraft carrier, that recently got dismantled at Alang in Gujarat?
  A. INS Vikrant. B. INS Viraat.
www.gradeup.co

  C. INS Arihant. D. INS Vikramaditya.


45. Why Institutes of Information Technology (IIIT) Laws Bill, 2020 was passes by the Parliament?

(I) IIT’s are envisaged to promote higher education and research in the field of Information Technology.

(II) The bill will confer the status of National Technology Laws (Amendment) Bill, 2020 was passed in Lok Sabha
on 20th March 2020.

(III) The bill will encourage IIITs to promote the study of information and technology in the country through their
innovative and quality methods.

(IV) This bill makes a fundamental right of every between the ages of 6 and 14 and specifies minimum norms in
elementary schools and colleges.
  A. I, II B. I, IV
  C. I, II, III D. I, II, III, IV
46. At which place, the Defence Research and Development Organization (DRDO), has successfully conducted the flight
test of ABHYAS – High Speed Expendable Aerial Target (HEAT).

  A. Balasore. B. Jodhpur.
  C. Bangalore. D. Madras.
47. Which Union ministry has approved the set-up of ten plastic parks in country under “Plastic Park Scheme”?

  A. Ministry of Environment, Forest and Climate Change. B. Department of Drinking Water and Sanitation.
  C. Ministry of Chemical and Fertilizers. D. Ministry of Home Affairs.
48. The name of the free flight artillery rocket system having a range of 37.5 km which is being manufactured by
Defence Research and Development Organization (DRDO)?

  A. Shaurya missile. B. Pinaka rocket.


  C. Nirbhay missile. D. Prahaar missile.
49. Which one among the following has been selected for “Shanti Swarup Bhatnagar Prize” in 2020 for outstanding
contributions to science and technology?

  A. Dr. Kayarat Saikrishnan. B. Dr. Ganesh Nagaraju.


  C. Dr. Sanjeev Das. D. Dr. Abhijit Mukherjee.
www.gradeup.co

50. On which date the “SAATHI” initiative was launched by the Ministry of Tourism?

  A. 27 September 2020, World Tourism Day. B. 7 April 2020, World Health Day.
  C. 8 May 2020, World Red Cross Day. D. 1 December 2020, World Acid Day.
51. The name of the India’s first museum on Ganga inaugurated by PM Narendra Modi?

  A. National Police Memorial Museum. B. Namami Gange.


  C. Ganga Avalokan. D. Sarnath Museum.
52. According to Economic Freedom of the World Report 2020, the position of India is?

  A. 79th position. B. 105th position.


  C. 53rd position. D. 92nd position.
53. On the occasion of Engineer’s Day 2020, the Department of Post released a postage stamp based on which of the
following?
  A. Dr. A.P.J Abdul Kalam. B. Indian Institute of Science.
  C. India-France: 50 years of space co-operation. D. Anti-Satellite Missile (A-SAT) launch.
54. Which one among the following, has announced its aim to put the world’s first hydrogen-powered commercial
aircraft into service by 2035?
  A. Dassault Aviation. B. Airbus.
  C. Spirit Aero System. D. United Aircraft Corporation.
55. What is the name of an android app that can analyze the COVID-19 Genome?
  A. Aarogya Setu App. B. Covid Watch App.
  C. Genopo App. D. NHS smartphone App.
56. Which one among the following is the new Managing Director (MD) and CEO of Petronet LNG Ltd. Appointed by
the director of Indian Oil Corp (IOC)?

  A. Akshay Kumar Singh. B. Sanjiv Singh.


  C. Sandeep Kumar Gupta. D. Sanjay Singh.
57. Which among the following formulated the “AYUSHMAN SAHAKAR” scheme on 19 October 2020?

  A. National Agriculture Infra Financing Facility. B. National Cooperative Development Corporation.


  C. National Committee on Food Processing. D. National Committee on Nutrition.
www.gradeup.co

58. According to World bank, the position/rank of India in the annual Human Capital Index in the year 2020 was?

  A. 45th rank. B. 96th rank.


  C. 22nd rank. D. 169th rank.
59. Which state has approved the New Information Technology Policy, 2020, which aims to enable the IT industry to
contribute 30% to India’s goal of becoming a trillion-dollar economy?

  A. Maharashtra B. Tamil Nadu.


  C. Karnataka. D. Gujarat.
60. Which one among the following state has been on top position in ease of doing business category, with respect to
Business Reform Action Plan (BRAP) Ranking?

  A. Telangana. B. Andhra Pradesh.


  C. Maharashtra. D. Karnataka.
61. Which one among the following, has implemented the “The Atal Bimit Vyakti Kalyan Yojana” which was introduced
in 2018?

B. Insurance Regulatory and Development Authority.


  A. Employee State Insurance Corporation. (ESIC)
(IRDAI)
  C. National Insurance Company Limited. (NICL) D. Life Insurance Corporation. (LIC)
62. Which country has discovered the large natural gas reserve off black sea?

  A. Saudi Arab. B. Iran.


  C. Russia. D. Turkey.
63. The new appointed Election Commissioner of India is?

  A. Ashok Lavasa. B. Sunil Arora.


  C. Rajiv Kumar. D. Achal Kumar Jyoti.
64. The India’s longest ropeway service was inaugurated on which of the following river?

  A. Ganga. B. Brahmaputra.
  C. Indus. D. Godavari.
65. Which country has established a BRICS innovation base for strengthening the cooperation among the 5-member
www.gradeup.co

countries?

  A. China. B. Russia.
  C. Brazil. D. India.
66. Which one among the following, has passed the resolution for the entire state to be included in the Schedule VI of
Indian Constitution?

  A. West Bengal. B. Arunachal Pradesh.


  C. Nagaland. D. Andaman & Nicobar.
67. Which Union ministry has launched a startup challenge contest “CHUNAUTI”?

  A. Ministry of Home affairs. B. Ministry of Cyber Defence.


  C. Ministry of Communications. D. Ministry of Electronics and Information Technology.
68. Which one among the following is the first Gulf nation that has signed the peace deal with Israel?
  A. Oman. B. UAE.
  C. Qatar. D. Kuwait.
69. Which bank has launched the ‘Shaurya KGC (Kisan Gold Credit)’, a loan product for the armed forces?

  A. State Bank of India (SBI). B. ICICI Bank.


  C. Yes Bank. D. HDFC Bank.
70. Which one among the following is the first organic state in India and the world?

  A. Sikkim. B. Uttarakhand.
  C. Tripura. D. Kerala.
71. Select the related word/letters/number from the given alternatives.

BCD : DEG : : FIJ : ?


  A. HKM B. JLN
  C. FGH D. KLN
72. Meghna drives 10 km towards South, takes a right turn and drives 6 Km. She then takes another right turn drives 10
km and stops. How far is she from the starting point?
  A. 16 km B. 6 km
  C. 4 km D. 12 km
www.gradeup.co

  E. None of these
73. If L x M means L is to the south of M;
L + M means L is to the north of M;
L % M means L is to the east of M;
L - M means L is to the west of M;

then in expression -  A % B + C - Z, Z is in which direction with respect to B?


  A. South-West B. South-East
  C. North-East D. North-West
  E. None of these
74. Direction: Study the following data carefully and answer the questions accordingly.

A certain number of people are sitting in a row facing south direction. R sits between P and Q. Six people are sitting
between T and Q, who sits to the left of W. Four people sit between R and W. Three people sit between Q and P. T
sits third to the right of W. G sits sixth to the right of R.

How many people sit in the row?


  A. Fifteen B. Twelve
  C. Thirteen D. Ten
  E. None of these
75. Who sits to the right of G?
  A. R B. Q
  C. T D. W
  E. Can’t be determined
76. How many people sit to the left of Q?
  A. Three B. Seven
  C. Four D. Five
  E. None of these
77. Who sits between W and R?
  A. P B. R
  C. T D. Q
  E. None of these
www.gradeup.co

78. How many people sit between T and R?


  A. Nine B. Ten
  C. Eight D. Seven
  E. None of these
79. Read the given statements and conclusions carefully. Assuming that the information given in the statements is true,
even if it appears to be at variance with commonly known facts, decide which of the given conclusions logically
follow(s) from the statements.

Statements:
I. All Maths are Physics.
II. All Physics are Chemistry.
III. No Chemistry are History.

Conclusions:
I. No Maths are History.
II. No Physics are History.
III. All Chemistry are Maths.
  A. None follows B. Only conclusion II follows.
  C. Only conclusion II and III follow. D. Only conclusion I and II follows.
80. In the following question below are given some statements followed by some conclusions. Taking the given
statements to be true even if they seem to be at variance from commonly known facts, read all the conclusions
and then decide which of the given conclusion logically follows the given statements.

Statements:
I. Some pens are pencils.
II. Some pens are erasers.

Conclusions:
I. Some pencils are erasers.
II. All erasers are pens
  A. Only conclusion (I) follows. B. Only conclusion (II) follows.
  C. Both conclusion follow. D. Neither conclusion (I) nor conclusion (II) follows.
www.gradeup.co

81. In the question two statements are given, followed by four conclusions, I, II, III and IV. You have to consider the
statements to be true even if it seems to be at variance from commonly known facts. You have to decide which of the
given conclusions, if any, follows from the given statements. 

Statements:
I. Some tumblers are LCD
II. Some LCD are radios.

Conclusions:
I. Some tumblers are radios.
II. Some radios are tumblers.
III. All the radios are LCD
IV. All the LCD are tumblers.
  A. Only (I) and (IV) B. Only (II) and (IV)
  C. Only I follow D. None of these
82. In the following question, two/three statements are given followed by four conclusions. You have to consider
the statements to be true even if they seem to be at variance from commonly known facts. You have to decide
which of the given conclusions, if any, follow from the given statements.

Statements:
I. All Trucks are Audi.
II. Some Scooters are Audi.

Conclusions:
I. All Trucks are Scooters.
II. Some Scooters are Trucks.
  A. Neither I nor II follows. B. Both I and II follows.
  C. Only II follows. D. Only I follows.
83. In a row of forty-five girls facing South, D is sixteenth from the right end. There are 8 girls between D and B. What
is B’s position from the left end of the row?
  A. Twenty-first B. Ninth
  C. Twentieth D. Data Inadequate
www.gradeup.co

  E. None of these
84. If A eats more than B but less than C, C eats more than D but less than E and F, then who eats in the largest quantity
among them?
  A. B B. E
  C. F D. Either E or F
85. Direction: Study the following data carefully and answer the questions accordingly.

A certain number of people are sitting around a circle facing the center. More than one person is sitting between D
and K. K sits fourth to the left of P. At most two and at least one person is sitting between J and P. Five people are
sitting between D and P, who is not sitting near to X. Seven people are sitting between S and J. Not more than four
people are sitting between S and P, from the right of S. At most fifteen people are sitting in this arrangement. D sits
third to the left of S and more than three people are sitting between D and X, from the left of D. X sits fifth to the left
of J.

How many people are sitting in this arrangement?


  A. 15 B. 11
  C. 14 D. 10
  E. None of these
86. How many people are sitting between D and J when counted from the left of D?
  A. One B. Five
  C. Three D. Two
  E. None of these
87. Who is sitting between S and X when counted from the right of S?
  A. D B. P
  C. J D. K
  E. None of these
88. What is the position of P with respect to K??
  A. Eighth to the left B. Third to the right
  C. Sixth to the left D. Fourth to the right
  E. None of these
89. Find the correct statement.
  A. D sits second to the right of J. B. S and K are not immediate neighbors.
www.gradeup.co

  C. X sits third to the left of P. D. K sits fourth to the right of D.


  E. None is correct
90. Direction: Study the following data carefully and answer the questions accordingly.
‘change wise income now’ is written as ‘Sa Ra Ta Na’,
‘income becomes cure issue’ is written as ‘Pa Ga Sa Fa’,
‘cure wise looking good’ is written as ‘Na Ma Pa Va’ and
‘Looking good change with’ is written as ‘Va Ma Ra Da’.

What could be the possible code for ‘wise good’?


  A. Ra Va B. Sa Ma
  C. Va Ta D. Ma Na
  E. None of these
91. What is the code for ‘Fa’?
  A. cure B. income
  C. issue D. becomes
  E. Either ‘issue’ or ‘becomes’
92. ‘Ra Ma Na’ represents which of the following?
  A. now good change B. good income change
  C. looking change wise D. wise change good
  E. Cannot be determined
93. If ‘the issue is worst’ is written as ‘Ia Ja Ga Ya’, then what will be the code for ‘becomes’?
  A. Ya B. Pa
  C. Ga D. Fa
  E. Cannot be determined
94. What is code for ‘looking’?
  A. Da B. Na
  C. Va D. Ma
  E. Either ‘Ma’ or ‘Va’
95. Pointing to a man Sabiya said, “His only brother is the father of my son’s father.” How is the man related to Sabiya?
  A. Father B. Brother
  C. Uncle D. Grand father
www.gradeup.co

96. V is the W’s sister, X is V’s father, Y is X’s mother, Z is W’s daughter. How is Y related to Z?
  A. Grandmother B. Great Grandmother
  C. Mother D. Sister
97. A woman pointing to a person said, “He is the son of my father’s only daughter.” How is the woman related to the
person?
  A. Sister B. Grandmother
  C. Mother D. Aunt
98. Direction: In these questions, a relationship between different elements is shown in the statements. The statements
are followed by two conclusions. Statements:
F>E<N<K=B>A>G; R=B<M

Conclusions:
I. R>G
II. F<B
  A. Only conclusion I is true. B. Only conclusion II is true.
  C. Either conclusion I or II is true. D. Neither conclusion I nor II is true.
  E. Both conclusion I and II is are true.
99. Direction: In the following question, some statements are followed by two conclusions (I and II). Assuming the
given statements to be true, find which of the two conclusions follow(s) the given statements and choose appropriate
answer choice.
Statements:
P ≥ K, B ≥ N, P ≤ N, T > B

Conclusions:
I. N < T
II. N > T
  A. Only conclusion I is true. B. Only conclusion II is true.
  C. Either conclusion I or conclusion II is true. D. Neither conclusion I nor conclusion II is true.
  E. Both conclusions I and II are true.
100. Direction: In the following question assuming the given statements to be true, find which of the conclusion among
given conclusions is /are definitely true and then give your answers accordingly.
www.gradeup.co

Statements:
H > J = K, K ≥ L, L > T, T < V
Conclusions:
I. K > T
II. L ≤ H
  A. Only conclusion I follows. B. Only conclusion II follows.
  C. Either conclusion I or II follows. D. Neither conclusion I nor II follows.
  E. Both conclusions I and II follow.
101. Direction: Study the following data carefully and answer the questions accordingly.

Seven people A, B, C, D, E, F, and G watched Houseful 4 movie on seven different days of the same week starting
from Sunday. F watched the movie before D. Three people watched the movie between A and E. Three people
watched the movie between G and B. C watched the movie after A. E watched the movie before G. Less than two
people watched the movie after B, who did not, watched the movie on Saturday.

Who watched the movie on Thursday?


  A. Can’t be determined B. B
  C. D D. C
  E. A
102. How many of them watched the movie before D?
  A. Four B. Three
  C. One D. Two
  E. None
103. Who watched the movie at last?
  A. E B. F
  C. A D. C
  E. None of these
104. F watched the movie on which of the following days?
  A. Friday B. Sunday
  C. Wednesday D. Tuesday
  E. None of these
105. Find the odd one.
www.gradeup.co

  A. F, E B. B, A
  C. F, A D. D, G
  E. B, D
106. Direction: Study the following data carefully and answer the questions accordingly.

Eight people A, B, C, D, E, F, G, and H are living in an eight-story building. The ground floor is numbered 1 and the
top floor is numbered 8. C lives on an even-numbered floor above the fifth floor. Two people live between D and
C. Three people live between G and D. Four people live between H and G. A lives below H but not on an even-
numbered floor. E lives three floors below B. F lives above A.

Who lives on the top floor?


  A. A B. B
  C. H D. E
  E. None of these
107. How many of them live between H and C?
  A. Five B. Three
  C. Four D. Two
  E. Can’t be determined
108. Who lives between D and E?
  A. F B. C
  C. B D. H
  E. Can’t be determined
109. Who lives on the second floor?
  A. F B. D
  C. A D. H
  E. None of these
110. Find the odd one.
  A. H B. B
  C. E D. C
  E. F
111. Direction: Read the given passage carefully and answer the questions that follow. 
www.gradeup.co

It wasn’t very long ago that a banana was just a banana – just a curved, yellow fruit. All you knew, if you bought a
bunch in 1986, was that they cost around 97p per kilo. You weren’t told if they were organic or pesticide-free. You
didn’t know if they came from Costa Rica or the Dominican Republic. And you certainly weren’t invited to worry
about the farmers who grew them – or if their children went to school, or whether their villages had clinics. You just
picked up your bananas and walked to the next aisle for your coffee or tea or chocolate, none the wiser about where
they came from either, or about the people who farmed them.

Back then, the countries that grew these commodities and many others were still known as the Third World, and the
habit of not caring about their farming conditions was a legacy of their colonial past. For centuries, trade propelled
the colonial project, and exploitation was its very purpose. The farmers of Asia, Africa and South America were
forced to raise the crops that the empire’s companies wanted, to work the crops in abject conditions, and to part with
them at ruinously low prices. In the last century, the empires melted away, but the trade remained lopsided – with the
imbalance now rationalized by the market, which deemed it “efficient” to pay farmers as little as possible. In the
1970s, a Ghanaian cocoa farmer often received less than 10 cents out of every dollar his beans earned on the
commodities market; as a proportion of the retail price of a chocolate bar, his take was smaller still. Child labor was
common. The chocolate companies prospered, and their customers shopped well; the farmers stayed poor.

Then, in the late 1980s, you began to hear more about these farmers, encountering their stories on television or in
newspapers or even on the labels of the packages you bought. The reasons were manifold. Environmental awareness
was on the rise. The prices of some commodities were crashing, placing agricultural incomes in even more acute
peril than usual. There had already been small groups pushing for more equitable trade: “little do-good shops
scattered in cities around Europe, selling products … bought at fair prices directly from small producers abroad”, as
one pioneer described it. By the early 1990s, these disparate initiatives began to coalesce into a larger international
struggle to radically reform our relationship with what we bought. Trade have long been unfair by design, but now
there was a growing movement to make consumers care about that unfairness, and even to help rectify it.

The crown jewel of this movement was Fairtrade International, an umbrella body formed in 1997 out of various
national chapters that had sprouted over the previous decade. Fairtrade was founded on the conviction that
consumers could make the marketplace more moral. The spine of Fairtrade’s philosophy has always been price:
simple, clean, the kind of measure that economists like to deal with. If companies pay farmers equitably, Fairtrade
believes, other benefits cascade out as well. Farmers can hire adult workers, rather than employing children; they can
send their kids to school and buy medicines; they can improve the yields of their farms by using better fertilizers.
Producers must meet a number of standards to qualify for Fairtrade: rules about labor conditions, for instance, or
www.gradeup.co

waste disposal. But for companies, the totality of their ethical responsibilities towards their producers is encapsulated
by price.

o, Fairtrade works by forming a kind of “virtuous triangle” of ethical business. It recruits farmers and farming
cooperatives as members, asking them to meet its standards; periodically, Fairtrade sends inspectors to these farms
around the world, ensuring they are still compliant. At another vertex of the triangle, Fairtrade enlists companies to
pay a minimum price, always set above market price, for commodities from these member farms and offers to certify
products made from such ethically sourced commodities. The final corner is the customer, who can, with a little
counsel, be galvanized to shop consciously, and to buy Fairtrade-certified products even if they cost a few pence
more.

But its two real triumphs are in the realm of ideas, not numbers. First, and most obviously, Fairtrade challenged the
entrenched model of the commodities business – the belief that a farmer’s lean income was just an unavoidable
reality of trade economics. Second, somewhat more stealthily, it cemented the notion that the modern corporation
would be ethical if only someone held up a lamp and showed it the way. Capitalism didn’t have to be feared; the
market would figure out its own checks and balances, through labeling agencies such as Fairtrade and Rainforest
Alliance, without having to be regulated by any higher authority. Persistent and egregious inequality could be solved
by deft pleats and tucks to the garb of trade, rather than by a full reconsideration of its fabric and seams.

Which of the following lines from the passage, justify the first paragraph of the passage?
B. In the late 1980s, you began to hear more about these
A. The prices of some commodities were crashing,
farmers, encountering their stories on television or in
  placing agricultural incomes in even more acute peril than
newspapers or even on the labels of the packages you
usual.
bought.
C. Farmers can hire adult workers, rather than employing
children; they can send their kids to school and buy D. The habit of not caring about their farming conditions
 
medicines; they can improve the yields of their farms by was a legacy of their colonial past.
using better fertilisers.
  E. None of the above
112. Which of the following can be inferred from the passage?

I. Digital and print media took the concern of farmers to a wide range of people.
II. Exploitation lay at the centre of colonial trade projects.
III. Fairtrade International was based on the belief that sellers could make the marketplace more fragile.
www.gradeup.co

  A. Both 1 & 2 B. Only 1


  C. Only 3 D. Both 1 & 3
  E. Only 2
113. With respect to the given passage, which of the following is true about “Fairtrade”?

I. It is based on the belief that the other benefits follow if the farmers are paid fairly.
II. The producers are not obliged to meet all the specified standards and the regulations are often relaxed for their
convenience.
III. The price paid to the producers constitutes the major ethical responsibility of the companies.
  A. Only I B. Only II
  C. Both I and III D. Both II and III
  E. All of these
114. Why did the people start hearing more about the farmers in the late 1980s?

I. Accelerating environmental awareness.


II. The decline in the price of many goods diminished agricultural incomes.
III. There had never been any group demanding equitable trade in the past, a factor that threw in a plethora of small
groups demanding the same in the 1980s.
  A. Only II B. Only III
  C. All I, II and III D. Both I and III
  E. Both I and II
115. The following sentence from the passage has been divided into four segments. One of these segments may be
grammatically incorrect. Find that part and mark the relevant option as the answer. If all the segments are correct,
“No error” is your answer?

Trade have long been unfair by design, (1)/ but now there was a growing movement to (2)/ make consumers
care about that (3)/ unfairness, and even to help rectify it. (4)
  A. 1 B. 2
  C. 3 D. 4
  E. No error
116. Direction: A sentence with one blank is given, indicating that something has been omitted. Choose the word that
best fits the blank appropriately. The detective knew the __________ scent had to be coming from a corpse.
  A. Amateur B. Beginner
www.gradeup.co

  C. Insular D. Malodorous
  E. None of these
117. Direction: In the following question, three sentences labelled (I), (II) and (III) have been given, followed by two
words. Each sentence carries a blank which may or may not be filled by the given words. From the given options,
choose the combination which represents the appropriate pair of sentence and word.

I. The government has come under _________ from all sides for cutting education spending.
II. Most wild animals won't _________ unless they are provoked or made to feel in danger.
III. In the _________, police found several boxes of obscene videotapes and illegal drugs.
(a) raid
(b) attack
  A. I-(b), II-(b) & III-(a) B. I-(a), II-(b) & III-(b)
  C. I-(b) & III-(b) D. I-(a), II-(b) & III-(a)
  E. None of the given combinations
118. Direction: A sentence with two blanks is given, each blank indicating that something has been omitted. Choose the
words that best fit in the given blanks making the sentence grammatically correct and meaningful. The move to allow
dumping of mercury __________ an outcry from residents of the area who fear that high levels of mercury will affect
their health and _______ ecologically sensitive forest area.
  A. resulted, insist B. provoked, destroy
  C. incited, determined D. activated, accept
  E. angered, believe
119. Direction: A sentence with one blank is given, indicating that something has been omitted. Choose the word that
best fits the blank appropriately. The city was _______ with rumours last night that the two had been executed.
  A. Agog B. Erudite
  C. Attrition D. Exclaimed
  E. None of these
120. Direction: Read the given passage carefully. Choose the most appropriate option from the given alternatives which
expresses the summary of the passage. The Internet Protocol (IP) is the principal communications protocol in the
Internet protocol suite for relaying datagrams across network boundaries. Its routing function enables
internetworking and essentially establishes the Internet. IP has the task of delivering packets from the source host to
the destination host solely based on the IP addresses in the packet headers. For this purpose, IP defines packet
structures that encapsulate the data to be delivered. It also defines addressing methods that are used to label the
www.gradeup.co

datagram with source and destination information. Historically, IP was the connectionless datagram service in the
original Transmission Control Program introduced by Vint Cerf and Bob Kahn in 1974; the other being the
connection-oriented Transmission Control Protocol (TCP).

i. Internet protocol is used to transfer data in the form of packets from source to destination on the basis of address in
the packet headers and addressing methods.
ii. Transmission Control Protocol defines packet structures that encapsulate the data to be delivered.
iii. Historically, when Vint Cerf and Bob Kahn introduced Transfer Control Protocol, it was connectionless datagram
service. Later it evolved to connection-oriented.
  A. Only i B. Only ii
  C. Only iii is correct D. Both i and iii
  E. None of these
121. Direction: Read the given passage carefully. Choose the most appropriate option from the given alternatives which
expresses the summary of the passage. The Great Depression was a severe worldwide depression that took place
mostly during the 1930s, beginning in the United States. The timing of the Great Depression varied across nations; in
most countries it started in 1929 and lasted until the late-1930s. It was the longest, deepest, and most widespread
depression of the 20th century. In the 21st century, the Great Depression is commonly used as an example of how far
the world's economy can decline.The Great Depression started in the United States after a major fall in stock prices
that began around September 4, 1929, and became worldwide news with the stock market crash of October 29, 1929
(known as Black Tuesday). Between 1929 and 1932, worldwide gross domestic product (GDP) fell by an estimated
15%. By comparison, worldwide GDP fell by less than 1% from 2008 to 2009 during the Great Recession.Some
economies started to recover by the mid-1930s. However, in many countries, the negative effects of the Great
Depression lasted until the beginning of World War II.

i. The Great Depression was a global level economic depression which was triggered with the market crash of 1929
and went on till late 1930s for most of the countries.
ii. Great Depression was a period of severe depression in teenagers and young adults which happened majorly due to
the uncertainty and disruption cause by the world war.
iii. The Great Depression started after World War - II and lasted for over a decade, and now is a widely used example
as to how much the world’s economy can decline.
  A. Only i B. Only ii
  C. Only iii D. Both i and iii
  E. None of these
www.gradeup.co

122. Direction: Read the given passage carefully. Choose the most appropriate option from the given alternatives which
expresses the summary of the passage. A black hole all by its lonesome doesn’t do much of anything, and even when
it eats some gas or dust, it’s very inefficient at converting its food to X-rays or any other form of radiation. But when
a black hole, or even a neutron star or a white dwarf, when they have a stellar companion—that is, a living star—that
star will shed gas and form a disk around the black hole. Those disks are very good at producing the X-rays
detectable by powerful technology.
A. When a black hole orbits together with a star, in a B. The slow process of augmenting the companion star
  formation known as a binary system, the black hole feeds produces a glowing disk of hot gas that can be seen by
on its companion, sucking the starlight into the shadows. powerful telescopes through X-rays.
D. It’s actually quite rare for a black hole to successfully
C. Without cannibalized stars to light the way, we can't lasso a star with its gravity and start chomping down on it,
 
see black holes with current technology. so there must be many more out there without a
companion than with one
E. Astronomers have detected evidence of a handful of the
  likely thousands of light-gobbling objects in the middle of
the galaxy.
123. Direction: Read the given passage carefully. Choose the most appropriate option from the given alternatives which
expresses the summary of the passage.
One of the key problems in tackling TB lies in ensuring that patients, once identified, actually take the prescribed
course of medication. To its credit, India does back the TB programme. Medicines are not really a problem, and are
provided free to patients. But the course of treatment is long: typically six months to a year.
A. When tackling TB, one of the biggest problems health
workers face is identifying the people who are sick and
  B. It is hard to detect TB
infectious early enough to prevent them from spreading
the disease.
C. Individuals who test positive for active tuberculosis
(TB) but do not initiate treatment present a challenge to D. A system is needed to make sure that TB patients
 
TB programmes because they contribute to ongoing completed their full course of treatment.
transmission within communities.
E. One advantage India did have was a public health
 
system and enough medicines for TB patients.
124. Which of the following refers to the idiom ‘Harp on the same tune’?
www.gradeup.co

  A. To be in the state of uncertainty B. To continue to hope for something even though it


seems unlikely to happen
  C. to delay or wait D. Keep on repeating the same thing
  E. None of these
125. Which of the following is the meaning of the idiom “Gall and wormwood”?
  A. Find something amusing B. something hateful
  C. To sacrifice each and everything D. To remain away
  E. None of these
126. Which of the following is the meaning of the idiom ‘In the bad books’ ?
  A. losing money B. in a bad situation
  C. Having fallen out of favour with someone D. uncomfortable or embarrassed
  E. None of these
127. Direction: In the following question, a part of the sentence is bold. Below the sentence alternatives to the bold part
are given at (A), (B), (C) and (D) which may improve the sentence. Choose the correct alternative. In case the given
sentence is correct, your answer is (E) i.e. No correction required. As soon as she opened the umbrella, a scorpion
fell about of it.
  A. fell up from B. fell off from
  C. fell off D. fell from off
  E. No correction required
128. Direction: In the following question, a part of the sentence is bold. Five alternatives to the bold part are given at (A),
(B), (C) and (D) which may improve the sentence. Choose the correct alternative. In case the given sentence is
correct, your answer is (E) i.e. No correction required. Mr. Jaitley said that a better spread of the monsoon meant that
the government could expect higher production of pulses this year, which should ease of their prices into the
market.
  A. ease its prices of the market B. ease their prices at the market
  C. easily its prices in the market D. ease their prices in the market
  E. No correction Required
129. Directions: Which of the phrases (1), (2), (3) and (4) given below the sentence should replace the phrase printed
inbold in the sentence to make tit grammatically correct? If the sentence is correct as it is given and no correction is
required, mark E. as the answer. The players have the right to roam free.
  A. Roaming free B. Travel free
www.gradeup.co

  C. Playing free D. Talk free


  E. No change required
130. Direction: In the given question, a part of the sentence is printed in bold. Below the sentence, alternatives to the
emboldened part are given as (A), (B), (C) and (D), which may help improve the sentence. Choose the correct
alternative out of the given five options. In case the given sentence is correct, your answer will be option (E), i.e.,
"No correction required".

Since it takes long to return a product to an online store than it does to a local shop, this becomes a deciding factor
for consumers on where to make the purchase.
  A. Since it took longer B. Since it takes longer
  C. As it takes long D. For it takes long
  E. No correction required
131. Direction: In each question, there are four statements A, B, C and D that have to be arranged in a logical order to
make a paragraph between 1 and 6. 1) The draw has been made, the schedule is complete, and the hyperbole pot is
beginning to simmer.
A) And it will stay that way till the final is played in Rio.
B) In travel terms, each of them can expect an increased profile as a possible holiday destination.
C) Yet you don’t have to be a football fan to grasp the power of this quadrennial sporting bonanza.
D) By 12,July when hosts Brazil kick off against Croatia in Sao Paulo, the FIFA World Cup will be inescapable
6) And with this in mind, this series of offer suggestions on how you can visit them in 2014 have started to throng. 
  A. ACBD B. BDCA
  C. DACB D. DCBA
  E. DBAC
132. Direction: The given sentences, when properly sequenced, form a coherent paragraph. Each sentence is labelled with
a letter. Choose the most logical order of the sentences from among the five given choices to construct a coherent
paragraph keeping 1 and 6 as the first and the last statements respectively. 1) One major cause of the current
weakened state of Indian banks is the level and volume of non-performing assets.
A) Yet, the fact remains that the banks allowed themselves to be pressurized into lowering their guard in one area of
business that is and should be their bread and butter of existence---risk assessment.
B) Descriptions such as ‘deceased portfolio’ and figures running into thousands of crores have all led to treating the
problem as a major one-time aberration requiring emergency treatment.
C) The casual explanations--- political interference, willful defaults, targeted lending and even fraudulent behavior
by banks—have some grain of truth in them.
www.gradeup.co

D) The problem has not been looked at in its proper perspective.


6) The response from the banks is to concentrate on somehow reducing the amount and number of accounts in this
category.
  A. DCAB B. DBCA
  C. BDCA D. ADCB
  E. BACD
133. Given below are four jumbled sentences. Pick the option that gives their correct order.

P) It was a nationwide revolt which started as a mutiny of Royal Indian Navy Soldiers working under the sovereignty
of the British Government.

Q) There were few causes that were making the path for revolt.

R) These causes included the racial discrimination of India naval staff.

S) The Royal Indian Navy Mutiny in India was started on 18th February 1946.
  A. SPQR B. PQRS
  C. SRQP D. PRSQ
134. Given below are four jumbled sentences. Pick the option that gives their correct order.

P) Not only this, lack of toilets also exposes one-third of the country’s women to the risk of sexual assault.

Q) Swachh Bharat Mission (SBM) or Clean India Mission was launched by Prime Minister of India, Narendra Modi.

R) As per estimates, poor hygiene and sanitation facilities cost India 600,000 lives annually due to diarrhea and other
such diseases.

S) It was launched on the 145th birth anniversary of Mahatma Gandhi on October 2, 2014, at Rajghat in New Delhi.
  A. RQSP B. RPQS
  C. SRQP D. PRSQ
135. Select the incorrectly spelt word.
  A. mississauga B. auspces
  C. torrent D. amenable
136. Select the correctly spelt word.
  A. unproveble B. suparstitious
www.gradeup.co

  C. differentiate D. jeckpot
137. Select the incorrectly spelt word.
  A. sophisticated B. specifically
  C. squeeze D. subsequant
138. Select the most appropriate synonym of the given word.

STARKLY
  A. innocently B. irreverently
  C. maturely D. obviously
139. Find a word that is the synonym of -

COMPASSIONATE
  A. pathetic B. aesthetic
  C. sympathetic D. warm
140. Select the most appropriate antonym of the given word.

HORRIFY
  A. affright B. petrify
  C. appal D. soothe
141. A man covers a total distance of 100 km on bicycle. For the first 2 hours, the speed was 20 km/hr and for the rest of
the journey, it came down to 10 km/hr. The average speed will be

  A. B.

  C. D.
142. A school bus running at 3/4th of its own speed reached the school in 1 hrs 40 minutes. If the bus would run at its own
speed, how much time it could save?
  A. 25 min B. 20 min
  C. 30 min D. 40 min
143. Monika spends 72% of her income. If her income increases by 20% and savings increase by 15%, then her
expenditure increases by: (correct to 1 decimal place)
  A. 19.8% B. 20.2%
  C. 21.9% D. 20.8%
www.gradeup.co

144. Two person A and B who can finish a work individually in 8 and 12 days agree to do the work but on the alternate
day. As B takes more time than A, he started first day and then A on the second day and so on. How long the work
will last?
  A. 5  days B.  days

  C. days D.  days
145. P started a piece of work and left after working for 4 days, then Q came and finished the work in 18 days. If P had
left the work after working for 6 days and Q would have finished it in 12 days, then in how many days P and Q can
working alone finish the entire work respectively?
  A. 15 and 20 B. 20 and 30
  C. 10 and 15 D. 10 and 30
146. A fan is listed at Rs.1500 and a discount of 20% is offered on the list price. What additional discount must be offered
to the customer now to bring the net price to Rs.1104?
  A. 8% B. 10%
  C. 15% D. 12%
147. The average marks of 40 students was found to be 68. If the marks of two students were incorrectly entered as 48 and
64 instead of 84 and 46 respectively, then what is the correct average?
  A. 68.35 B. 68.15
  C. 68.45 D. 68.25
148. A started a business with Rs. 21000. Later A is joined by B with Rs. 36000. At the end of a year they divided the
share equally. After how many months B joined the business?
  A. 7 months B. 5 months
  C. 6 months D. 4 months
149. The present ages of P and Q are in the ratio 2 : 3. Ten years ago, this ratio was 3 : 5. The present ages of P and Q are
respectively :
  A. 44 years, 66 years B. 38 years, 57 years
  C. 40 years, 60 years D. 34 years, 51 years
150. A lent Rs. 5000 to B for 2 years and Rs. 3000 to C for 4 years on simple interest at the same rate of interest and
received Rs. 2200 in all from both of them as interest. The rate of interest per annum is:
  A. 9% B. 10%
  C. 11% D. 12%
www.gradeup.co

Solutions
1. A
Sol. Principle clearly states that granting right for the enjoyment shall be up to reasonable limit.
Cable wires were attached at a reasonable distance hence going strictly by the principle, A
would be the right answer.
2. A
Sol. Principle talks about forcibility of an act. Ram was totally unaware about the content of the
package hence A would be the right answer.
3. A
Sol. Principle talk about the given warning against the trespasser of the land. Warning was
sufficient and clear. Hence, factually the warning was breached by the trespasser hence A is
the right answer.
4. B
Sol. As per the legal recognition and principles, in the application of ex post facto cases, the rule of
beneficial interpretation in applied hence as per the fact, B would be the right answer.
5. C
Sol. Section 497 clearly exempts the wife from the penal liability hence, as per the present set of
facts, B would be the right answer.
6. A
Sol. As the principle clearly states the extent of the applicability of offence of abetment, A would be
the right answer as it’s the same act that gave birth to the another subsequent act.
7. A
Sol. Principle is clear from the point of view of intention. As per the present set of facts, the
intention was clear to cause a death hence, going by the principle, A would be the right
answer.
8. B
Sol. Principle is clear about the age being 7 years. The offence is done by 6 years old hence going
strictly with the principle; B would be the right answer.
9. D
www.gradeup.co

Sol. As the principle is clear that it does not take consideration of trifles issues, as per present set
of facts, D would be the right answer.
10. B
Sol. The act of a nurse is clearly the act of ignorance of fact, hence going strictly by the principle, B
would be the right answer.
11. B
Sol. The act of a nurse is clearly the act of ignorance of fact, hence going strictly by the principle, B
would be the right answer.
12. A
Sol. Principle clearly talk about voluntarily drunkness and in the facts it is established that its an act
of voluntarily drunkness hence A would be the right answer.
13. C
Sol. As the parties did not understand the same thing in a same manner, C would be the right
answer going strictly with the principle given.
14. C
Sol. The suit is maintainable as the since it is dispute arising out of a contract between a carrier
which is the subsidiary of the state.
15. A
Sol. The result of death arises out of the act of stabbing hence as per the present set of facts, A
would be the right answer.
16. A
Sol. As per the facts, the coat was in the possession of the dry cleaner hence if it its belonged to A,
the possession was important hence A would be the right answer.
17. A
Sol. As per the facts, the coat was in the possession of the dry cleaner hence if it its belonged to A,
the possession was important hence A would be the right answer.
18. D
Sol. D would be the right answer as the consent gathered for the enforceability of the contract is by
threat hence it will not be considered as a willful and consent and thus the contract would be
www.gradeup.co

considered negated.
19. B
Sol. As per the provisions of the contract act, the consideration is the key element for any contract
to be made enforceable. In the present set of facts, there was an agreement made without any
consideration hence agreement stands void and B would be the right answer.
20. C
Sol. As per the principle stated, it is quite clear that the appointment of the judges of Supreme
Court and the High Court is under the purview of the president. In the present set of facts, the
appointment was made by the prime minister hence appointment of Mr. Adarshnath and Mr.
Gargik is void ab initio.
21. B
Sol. As the words expressed by Mr. Sinha is gross and straightforwardly joining Mr. Rahul from
individual front and are sufficiently adequate to drop down his picture before overall population
henceforth under Section 499 of IPC.
22. B
Sol. The right to be certified by ICSI is not absolute as it is a professional course and in
professional courses such certifications have certain criteria’s which are ought to be cleared,
hence B would be the right answer.
23. D
Sol. D would be the right answer as the consent gathered for the enforceability of the contract is by
threat hence it will not be considered as a willful and consent and thus the contract would be
considered negated.
24. B
Sol. As per the provisions of the contract act, the consideration is the key element for any contract
to be made enforceable. In the present set of facts, there was an agreement made without any
consideration hence agreement stands void and B would be the right answer.
25. C
Sol. As per the principle stated, it is quite clear that the appointment of the judges of Supreme
Court and the High Court is under the purview of the president. In the present set of facts, the
www.gradeup.co

appointment was made by the prime minister hence appointment of Mr. Adarshnath and Mr.
Gargik is void ab initio.
26. C
Sol. The correct answer is C as the facts suggest that opening the training classes for CS might
cause damage to Mr. Ketan but there is legal injury involved in it hence C would be the right
answer.
27. A
Sol. As per the facts given and abiding by the principle, the company if formed under any of the
provisions of Companies Act, 1956 would be called as a company hence A would be the right
answer.
28. A
Sol. As per the given set of facts and reading the same with principle given, as Mr. Shyam was
delegated by ABC Ltd. hence there was established Master Servant Relationship hence for
the misdeed of Mr. Shyaam, ABC Ltd would be held liable and A would be the right answer.
29. A
Sol. In the given principle it talked about only the qualification for being the minister nut not about
the appointment of the minister hence abiding by the principle given, A would be the right
answer.
30. C
Sol. Answer C as having six ferocious dogs indicates the act of creating unnecessary nuisance
against the general public hence Mr. Surendra would within his purview of initiating the legal
proceeding against Mr. Sunil.
31. B
Sol. The principal mentions that promisors may compel other joint promisors to pay their share in
joint promise. In accordance with the same even if Bhupendra gives the whole of the amount
Bhupendra may recover the respective shares from Amitabh and Chintan.
32. B
Sol. Here, the basis of judgement is that presence of contrary intention. A contrary intention is
visible from the facts narrated by the above question. Since there is a contrary intention
www.gradeup.co

Bhupendra may not be compelled by other to for recovery of the amount.


33. B
Sol. As per principle a representative of the person who dies maybe one from home share of the
promises who died may be recovered. Hans ordinarily Amitabh and Bhupendra Mein pay their
shares while the share of Chintan may be recovered from his representative, therefore the
distribution in option to stands as the most appropriate one.
34. C
Sol. In most commercial contract two or more parties typically undertake to perform specific
application vis-a-vis Each Other such application could be like reciprocal promises that are
promises which form the part of inter consideration for each other in other. Hence C would be
the right answer.
35. B
Sol. Here the consent of marriage that was required to be taken by Q was taken after the marriage.
With strict application of the principal it is visible that the same was valid since it was a
condition to the contract which was complied with after the happening of the event of marriage
to which this condition of taking consent was attached.
36. A
Sol. The agreement to give Rs. 4000/- for giving the same to Randeep is an agreement collateral
to the wagering agreement. Horse racing may be considered a wagering agreement but that of
giving the amount to Randeep is not a wager but collateral one. Hence, since the agreement is
valid the bank can recover the amount.
37. B
Sol. The promise of discovering Treasure by magic made by P to Q will come within the Ambit of
an impossible promise as has been mentioned by the principal. In accordance with the
principal law does not enforce a promise that is impossible and therefore the law will not
enforce this promise here.
38. A
Sol. Bailment has not taken place here as goods were never delivered by Ram to the police and
also can they never be disposed of according to his directions. The absence of delivery itself
www.gradeup.co

clears the possibility of bailment here.


39. B
Sol. Consideration cannot be unlawful. The same has been stated in the principle. In the given set
of facts, Consideration was to kill Priya which was an offence and hence unlawful. So, B would
be the right answer.
40. A
Sol. Going exactly by the principle. Offer of 3 lakh was superseded by the counter offer of Rs 2.5
lakhs. Hence, in order to validate the offer, new offer offer of Rs 3 lakh have to be made. So, A
would be the correct answer.
41. A
Sol. The minimum support price (MSP) is an agriculture product price that is set by the
Government of India in order to purchase directly from the farmer. The MSP is fixed on the
recommendations of the Agriculture Costs and Prices Commission.
The Union Government has increased the Minimum Support Price (MSP) of Rabi crops and
wheat with an increase by Rs 50 per quintal. The other crops for which MSP has been
increased include Grams, Mustard, Safflower and Masoor.
42. D
Sol. The “Ghar Tak Fibre” acheme is likely to connect 45,945 villages of Bihar through optical fibre
internet service. The project will lead digital services including e-Education, e-Agriculture, Tele-
Medicine, Tele-law and other social security schemes in Bihar ensuring easy access to all
state natives.
The “Ghar Tak Fibre” project would entail implementation of 1 Wi-Fi and 5 free of cost
connections to government institutions including primary schools, Anganwadi Centers, Aasha
workers and Jeevika Didi. This project will be jointly executed by the Department of Telecom
(DoT), Ministry of Electronics & Information Technology and Common Service Centers (CSC).
43. C
Sol. The Union Government is planning to construct a high-altitude at 16,730 feet high Shinku La in
Himachal Pradesh.
www.gradeup.co

The National Highways and Infrastructure Development Corporation Limited (NHIDCL) has
speeded up the detailed project report (DPR) work on the 13.5 km Shinku La Tunnel
connecting the Union Territory of Ladakh and Lahaul and Spiti district of Himachal Pradesh.
With the construction of the tunnel under Shinku La, the inhabitants of around 15-odd villages
of Zanskar valley will be all-weather connected as the valley remains cut off due to heavy
snowfall during the winter.
44. B
Sol. INS Viraat, the world’s longest serving Aircraft Carrier, is to be dismantled at Alang in Gujarat
and sold as scrap. It was decommissioned in 2017 after 30 years of service with Indian Navy.
The ship was commissioned into the Royal Navy as HMS (Her Majesty’s Ship) Hermes in
1959. Then, it was commissioned into the Indian Navy in 1987 as INS (Indian Naval Ship)
Viraat, which means enormous.
The ship was capable of a maximum speed of 28 knots and has weight of 28,700 tons.
Presently, INS Vikramaditya is the sole aircraft carrier operated by the Indian Navy.
45. C
Sol. The Parliament has passed the Indian Institutes of Information Technology Laws (Amendment)
Bill 2020. The bill seeks to give 5 IITs in Public Private Partnership (PPP) mode – IIIT Bhopal,
IIIT Agartala, IIIT Surat, IIIT Bhagalpur and IIIT Raichur, a tag of Institutions of National
importance.
Currently there are 20 IIITs which are designated at institutions of national importance. With
the addition of these 5 institutes, the total number would increase to 25.
The Institutions of National importance would have the power to give degrees and categories
them as B. Tech, M. Tech or Ph.D. degree, which is currently issued by a University of
National Importance.
46. A
Sol. The DRDO has successfully conducted the flight test of the indigenously-designed ABHYAS
High-speed Expendable Aerial Target (HEAT) in Balasore (Odisha).
ABHYAS is designed & developed by Aeronautical Development Establishment (ADE),
DRDO.
www.gradeup.co

The radar cross section (RCS) of ABHYAS and its visual infrared signatures can be used to
simulate a variety of aircrafts for air-defense weapon practices.
Two demonstrator vehicles of ABHYAS cleared all the evaluation parameters like 5 km flying
altitude, vehicle speed of 0.5 mach (half the speed of sound), endurance of 30 minutes etc.
47. C
Sol. The Ministry of Chemical and Fertilizers has approved to set up ten plastics parks in the
scheme which was launched in 2019.
The Plastic Park scheme has been launched with an aim to increase the competitiveness and
value addition in the plastic processing industry through research and development in the field.
The plastic parks will be set-up in Madhya Pradesh, Assam, Tamil Nadu, Odisha, Jharkhand,
Chhattisgarh and Uttarakhand.
The Government of India will share Rs 40 crore per project and rest of the cost will be borne
by the state government.
48. B
Sol. Defence Research and Development Organization (DRDO) has recently started production of
Pinkaka rocket system for the Indian Armed forces. It is a free flight artillery rocket system
having a range of 37.5 km.
Pinaka rockets are launched from a multi-barrel rocket launcher which has a capability to
launch 12 rockets in 44 seconds. The weapon system is designed and developed by Pune-
based DRDO lab, Armament Research and Development Establishment (ARDE).
49. D
Sol. Dr. Abhijit Mukherjee, an Associate Professor from IIT Kharagpur has been selected for
“Shanti Swarup Bhatnagar” Prize. This Prize is given each year for outstanding contributions
to science and technology.
The award is named after Dr. Shanti Swarup Bhatnagar, the founder Director of the Council of
Scientific & Industrial Research (CSIR).
50. A
Sol. SAATHI is an initiative of the Ministry of Tourism with Quality Council of India to assist the
hospitality industry to continue to operate safely and thereby instill confidence among the staff,
www.gradeup.co

employees and the guests about the safety of the hotel and units.
It was launched on the occasion of World Tourism Day on 27 September 2020.
51. C
Sol. PM Narendra Modi has recently inaugurated Ganga Avlokan Museum – India’s first museum
on Ganga. It has been dedicated to showcase the biodiversity, rejuvenation and culture in
Ganga river.
The museum is located at Chandi Ghat, Haridwar. I t has been launched under Namami
Gange Program, which is mission to achieve effective abatement of pollution and conservation
and rejuvenation of National River Ganga. The mission is operated under Ministry of Jal Shakti
and implemented by the National Mission for clean Ganga.
52. B
Sol.
India witnessed a sharp fall of 26 spots and landed to the 105th position in recently launched
Global Economic Freedom Index 2020 report. As per the last year rankings, India was at the
79th spot.
It is an annual index created in 1995 by the Heritage Foundation and the Wall Street Journal to
gauge the degree of economic freedom in the world’s nations.
The main purpose of the report is to measure economic freedom, the economic security of
privately owned property, levels of personal choice, the ability for business players to enter
markets, rule of law, among other by analyzing policies and institutions of 162 countries and
territories.
53. D
Sol. On the occasion of Engineer’s Day, the Department of Post released a stamp on India’s first Anti-Satellite Missile
(A-SAT) launch.

Defence Research and Development Organization (DRDO) has successfully conducted an Anti-Satellite (A-SAT)
missile test under ‘Mission Shakti’ from Dr. A.P.J Abdul Kalam Island in Odisha on 27th March 2019. It has
capability of destroying enemy satellite in space.
54. B
Sol. Airbus has announced its aim to put the world’s first hydrogen-powered commercial aircraft
www.gradeup.co

into service by 2035. Airbus’ hydrogen technology will be demonstrated in three concept
aircrafts called Zeroe: a turboprop (up to 100 passengers), a turbo fan (100 to 200
passengers) and a blended wing body (up to 200 passengers).
Hydrogen is chosen as it produces zero emission.
55. C
Sol. A new genome sequencing app has been developed by scientist that can analyze the genome
of SARS-CoV-2 within minutes. The app named Genopo is available for download on Google
Play Store.
According to a study published in the journal communications Biology, Genopo app took less
than 30 minutes to determine the complete genome sequence of the human coronavirus.
56. A
Sol. Indian Oil Corporation director-pipelines Akshay Kumar Singh will be the new MD and CEO of
the country’s biggest gas importer, Petronet LNG Ltd.
Akshay Kumar Singh replaces Prabhat Singh who completed his five-year term on September
13, 2020. He was executive director at state-owned gas utility GAIL India Ltd before joining the
IOC board in 2018.
57. B
Sol. Union Agriculture Minister Narendra Singh Tomar launched AYUSHMAN AHAKAR scheme on
19 October, 2020.
AYUSHMAN SAHAKAR scheme has been formulated by the National Cooperative
Development Corporation (NCDC).
The scheme aims at bringing transformation in the health service primarily in the rural areas. It
provides interest subvention of one per cent to women majority cooperatives.
58. D
Sol. The World Bank released the annual Human Capital Index for the year 2020, in which India

was ranked at 169th position.


The Human Capital Index includes education and health data from 174 countries. It covers
98% of the world population. It provides pre-pandemic baseline on the education and health.
59. C
www.gradeup.co

Sol. Karnataka Cabinet has approved a new Information Technology Policy, 2020.
The new policy will boost IT penetration and innovation, distributed labor force and evolve a
Cyber security policy. The policy is to be executed between 2020 and 2025.
The main aim of the policy is to enable the IT industry to contribute 30% to India’s goal of
becoming a trillion-dollar economy.
60. B
Sol.
The Union Government has announced the 4th edition of Business Reform Action Plan
(BRAP) ranking of states. Andhra Pradesh retained its top position in Ease of Doing Business
category.
The lowest 3 ranks in the list belong to Odisha, Sikkim and Tripura. The top 5 states under
State Reform Action Plan 2019 are:
1. Andhra Pradesh.
2. Uttar Pradesh.
3. Telangana.
4. Madhya Pradesh.
5. Jharkhand.
61. A
Sol. The Employee State Insurance Corporation (ESIC) extended the unemployment benefit
scheme Atal Bimit Vyakti Kalyan Yojana till June 2021.
The Atal Bimit Vyakti Kalyan Yojana is being implemented by Employee State Insurance
Corporation. The scheme was approved under the Employees State Insurance Act, 1948.
This scheme was introduced in 2018. The scheme provides 25% of average per day earning
in the previous four contribution periods. The scheme pays cash directly to the bank account
of insured persons in case of unemployment.
Under the scheme, the workers can draw 47% of total contributions towards ESIC after being
unemployed for at least three months.
62. D
Sol. Turkey has found significant gas resources in the Black Sea. This would help ease the
country’s dependence on imports. The amount of gas discovered is 320 billion cubic meters, a
www.gradeup.co

sum industry analyst said was notable but not a ‘game-changer’ that might turn the country
into a regional energy hub or materially alter its financial fortunes.
63. C
Sol. The former Finance Secretary Rajiv Kumar has been appointed as an Election Commissioner.
He succeeded Ashok Lavasa who resigned being appointed as a Vice-President of Asian
Development Bank. The Election Commissioner of India is an autonomous constitutional
authority responsible for administering Union and State election process in India.
64. B
Sol. The longest river ropeway of India was inaugurated over the Brahmaputra river connecting
Guwahati city and North Guwahati in Assam.
The ropeway has been operated from Kachari Ghat in Guwahati to Dol Govinda temple in
North Guwahati. It passes over the mid-river Peacock Island that houses Umananda, a
medieval Shiva temple. It thus, cuts travel time between the two banks to 8 minutes.
65. A
Sol. China is going to establish a BRICS innovation base for strengthening the cooperation among
the 5-member countries group including India in technical sectors like 5G, Artificial Intelligence
and digital economy, industry and information technology.
BRICS group is composed by the 5 major emerging economies: Brazil, Russia, India, China
and South Africa (BRICS) formed in 2009.
66. B
Sol. Arunachal Assembly passes a resolution for the entire state to be included in the sixth
Schedule of the Indian constitution. It provides protection of tribal people’s right in the
Northeast states.
The sixth Schedule of the constitution of the constitution provides for the administration of
tribal areas in Assam, Tripura, Meghalaya and Mizoram.
It safeguards the right of tribal populations in these states. It was included based on the
reports of the Bordoloi Committee that was formed by the Constituents Assembly of India.
67. D
Sol. Union Ministry of Electronics and Information Technology has launched a startup challenge
www.gradeup.co

“CHUNAUTI” to promote startup indulged in software products.


“CHUNAUTI” is an acronym for “Challenge Hunt Under NGIS for Advanced Uninhibited
Technology Intervention”.
The objective of CHUNAUTI is to find the products as well as solutions to address the
challenges faced during the COVID-19 pandemic.
68. B
Sol. According to the peace agreement, the United Arab Emirates (UAE) will formally recognize the
state of Israel, while the latter would halt its plans to annex parts of the occupied West Bank of
Palestine.
The UAE will be only the third Arab country – and the first in the Gulf – to recognize Israel.
There will be formal diplomatic and business contacts, flight services will be launched and
both countries could cooperate more directly in regional diplomacy.
69. D
Sol. HDFC Bank has launched ‘Shaurya KGC (Kisan Gold Credit)’, a loan product for the armed
forces. This is the first of its kind scheme meant for the army personnel in the country and is
based on the Kisan Credit Card guidelines of the government.
This credit card comes with a life cover of Rs 10 lakhs.
It will provide finance for the agriculture requirements such as the production of crops, post-
harvest maintenance and consumption needs, purchase of farm machinery, irrigation
equipment, etc.
The loan facility can be availed by visiting HDFC Bank branch or through the e-Kisan Dhan
app of the bank.
70. A
Sol.
The Ministry of Agriculture and Farmers; welfare announced that India ranks 1st in Organic
Farming.
India also ranks 9th in terms of areas under Organic Farming.
Sikkim was the 1st state in the world to become completely organic. The other states that have
similar targets include Uttarakhand and Tripura.
71. A
www.gradeup.co

Sol. As,
B+2=D
C+2=E
D+3=G
Similarly,
F+2=H
I+2=K
J+3=M
Hence, option A is the correct answer.
72. B
Sol.

From the figure, it is clear that Meghna is 6 Km far from the starting point.
73. B
Sol. Z is in South East direction with respect to B

74. B
Sol. 1) G sits sixth to the right of R.

2) Four people are sitting between R and W.


www.gradeup.co

3) T sits third to the right of W.

4) Six people are sitting between T and Q, who is sitting to the left of W.

5) Three people are sitting between Q and P.


www.gradeup.co

6) R is sitting between P and Q.

(Here, case 2 will be eliminated)

Therefore, option B is the correct answer.


75. C
Sol. 1) G sits sixth to the right of R.

2) Four people are sitting between R and W.


www.gradeup.co

3) T sits third to the right of W.

4) Six people are sitting between T and Q, who is sitting to the left of W.

5) Three people are sitting between Q and P.


www.gradeup.co

6) R is sitting between P and Q.

(Here, case 2 will be eliminated)

Therefore, option C is the answer.


76. C
Sol. 1) G sits sixth to the right of R.

2) Four people are sitting between R and W.


www.gradeup.co

3) T sits third to the right of W.

4) Six people are sitting between T and Q, who is sitting to the left of W.

5) Three people are sitting between Q and P.


www.gradeup.co

6) R is sitting between P and Q.

(Here, case 2 will be eliminated)

Therefore, option C is the correct answer.


77. D
Sol. 1) G sits sixth to the right of R.

2) Four people are sitting between R and W.


www.gradeup.co

3) T sits third to the right of W.

4) Six people are sitting between T and Q, who is sitting to the left of W.

5) Three people are sitting between Q and P.


www.gradeup.co

6) R is sitting between P and Q.

(Here, case 2 will be eliminated)

Therefore, option D is correct.


78. D
Sol. 1) G sits sixth to the right of R.

2) Four people are sitting between R and W.


www.gradeup.co

3) T sits third to the right of W.

4) Six people are sitting between T and Q, who is sitting to the left of W.

5) Three people are sitting between Q and P.


www.gradeup.co

6) R is sitting between P and Q.

(Here, case 2 will be eliminated)

Therefore, option D is the correct answer.


79. D
Sol. Minimum Possible diagram is-
www.gradeup.co

Conclusions:

I. No Maths are History.( It follows as its obvious from the above diagram.)

II. No Physics are History. ( It follows as its obvious from the above diagram.)

III. All Chemistry are Maths. ( It does not follow as its just a possibility, not surety.)

So, Only conclusion I and II follows .

Hence, option D is the Correct answer.


80. D
Sol. Possible Venn-diagram of the given sentences is –

I. Some pencils are erasers. - it is not a definite case, hence false.


II. All erasers are pens - it is not a definite case, hence false.
Thus, neither conclusion (I) nor (II) follows.
www.gradeup.co

Hence, option D is the correct answer.


81. D
Sol. Here two possibilities emerge based on statements I and II.

Conclusion I:- It cannot be said with certainty that some tumblers are radios.
Conclusion II: - It cannot be said with certainty that some radios are tumblers.
Conclusion III:- It cannot be said with certainty that all the radios are LCD.
Conclusion IV:- It cannot be said with certainty that all the LCD are tumblers.
Therefore, none of the conclusion follows.

Hence, option D is the correct answer.


82. A
Sol. Here two possibilities emerge based on statement I and II.

Conclusion I:- It cannot be said with certainty that all trucks are scooters.
Conclusion II: - It cannot be said with certainty that some scooters are trucks.
Hence, neither conclusion I nor II follows.

Hence, option A is the correct answer.


83. D
Sol. Girls are facing south.
www.gradeup.co

It is not clear B is to the left or right of D.


Hence Option D is correct
84. D
Sol. Given that, A eats more than B but less than C it means C> A > B_________(1)

C eats more than D but less than E and F it means (E and F) > C>D____________(2)

From (1) and (2),

(E and F) > C > A > B

We know C eats more than D and (E and F) > C, therefore, either E or F eats in the largest quantity among them.

Hence, option D is the correct answer.


85. C
Sol. 1) X sits fifth to the left of J.

2) Seven people are sitting between S and J.


www.gradeup.co

3) D sits third to the left of S and more than three people are sitting between D and X, from the left of D.

4) Five people are sitting between D and P, who is not sitting near to X.

5) At most fifteen people are sitting in this arrangement.

6) At most two and at least one person is sitting between J and P.

7) Not more than four people are sitting between S and P, from the right of S.
www.gradeup.co

8) K sits fourth to the left of P.

9) More than one person is sitting between D and K.

(Here, case 2 will be eliminated)

Therefore, option C is the correct answer.


86. D
Sol. 1) X sits fifth to the left of J.

2) Seven people are sitting between S and J.


www.gradeup.co

3) D sits third to the left of S and more than three people are sitting between D and X, from the left of D.

4) Five people are sitting between D and P, who is not sitting near to X.

5) At most fifteen people are sitting in this arrangement.

6) At most two and at least one person is sitting between J and P.

7) Not more than four people are sitting between S and P, from the right of S.
www.gradeup.co

8) K sits fourth to the left of P.

9) More than one person is sitting between D and K.

(Here, case 2 will be eliminated)

Therefore, option D is the correct answer.


87. D
Sol. 1) X sits fifth to the left of J.

2) Seven people are sitting between S and J.


www.gradeup.co

3) D sits third to the left of S and more than three people are sitting between D and X, from the left of D.

4) Five people are sitting between D and P, who is not sitting near to X.

5) At most fifteen people are sitting in this arrangement.

6) At most two and at least one person is sitting between J and P.

7) Not more than four people are sitting between S and P, from the right of S.
www.gradeup.co

8) K sits fourth to the left of P.

9) More than one person is sitting between D and K.

(Here, case 2 will be eliminated)

Therefore, option D is the correct answer.


88. D
Sol. 1) X sits fifth to the left of J.

2) Seven people are sitting between S and J.


www.gradeup.co

3) D sits third to the left of S and more than three people are sitting between D and X, from the left of D.

4) Five people are sitting between D and P, who is not sitting near to X.

5) At most fifteen people are sitting in this arrangement.

6) At most two and at least one person is sitting between J and P.

7) Not more than four people are sitting between S and P, from the right of S.
www.gradeup.co

8) K sits fourth to the left of P.

9) More than one person is sitting between D and K.

(Here, case 2 will be eliminated)

Therefore, option D is the correct answer.


89. D
Sol. 1) X sits fifth to the left of J.

2) Seven people are sitting between S and J.


www.gradeup.co

3) D sits third to the left of S and more than three people are sitting between D and X, from the left of D.

4) Five people are sitting between D and P, who is not sitting near to X.

5) At most fifteen people are sitting in this arrangement.

6) At most two and at least one person is sitting between J and P.

7) Not more than four people are sitting between S and P, from the right of S.
www.gradeup.co

8) K sits fourth to the left of P.

9) More than one person is sitting between D and K.

(Here, case 2 will be eliminated)

Therefore, option D is the correct answer.


90. D
Sol. ‘change wise income now’ is written as ‘Sa Ra Ta Na’,

‘income becomes cure issue’ is written as ‘Pa Ga Sa Fa’,

‘cure wise looking good’ is written as ‘Na Ma Pa Va’,


www.gradeup.co

‘Looking good change with’ is written as ‘Va Ma Ra Da’.

change = Ra

wise = Na

income = Sa

now = Ta

cure = Pa

becomes/issue = Ga/Fa

looking/good = Ma/Va

with = Da

Therefore, ‘Ma Na’ could be the possible code for ‘wise good’.
91. D
Sol. ‘change wise income now’ is written as ‘Sa Ra Ta Na’,

‘income becomes cure issue’ is written as ‘Pa Ga Sa Fa’,

‘cure wise looking good’ is written as ‘Na Ma Pa Va’,

‘Looking good change with’ is written as ‘Va Ma Ra Da’.

change = Ra

wise = Na

income = Sa

now = Ta

cure = Pa

becomes/issue = Ga/Fa
www.gradeup.co

looking/good = Ma/Va

with = Da

Therefore, the code for ‘Fa’ could be ‘issue’ or ‘becomes’.


92. D
Sol. ‘change wise income now’ is written as ‘Sa Ra Ta Na’,

‘income becomes cure issue’ is written as ‘Pa Ga Sa Fa’,

‘cure wise looking good’ is written as ‘Na Ma Pa Va’,

‘Looking good change with’ is written as ‘Va Ma Ra Da’.

change = Ra

wise = Na

income = Sa

now = Ta

cure = Pa

becomes/issue = Ga/Fa

looking/good = Ma/Va

with = Da

Clearly, ‘Ra Ma Na’ could be the code for either ‘looking change wise’ or ‘wise change good’.

Therefore, it can’t be determined exactly.


93. D
Sol. ‘change wise income now’ is written as ‘Sa Ra Ta Na’,

‘income becomes cure issue’ is written as ‘Pa Ga Sa Fa’,


www.gradeup.co

‘cure wise looking good’ is written as ‘Na Ma Pa Va’,

‘Looking good change with’ is written as ‘Va Ma Ra Da’.

‘the issue is worst’ written as ‘Ia Ja Ga Ya’

change = Ra

wise = Na

income = Sa

now = Ta

cure = Pa

becomes/issue = Ga/Fa

looking/good = Ma/Va

with = Da

Therefore, the code for ‘becomes’ is ‘Fa’.


94. D
Sol. ‘change wise income now’ is written as ‘Sa Ra Ta Na’,

‘income becomes cure issue’ is written as ‘Pa Ga Sa Fa’,

‘cure wise looking good’ is written as ‘Na Ma Pa Va’,

‘Looking good change with’ is written as ‘Va Ma Ra Da’.

change = Ra

wise = Na

income = Sa

now = Ta
www.gradeup.co

cure = Pa

becomes/issue = Ga/Fa

looking/good = Ma/Va

with = Da

Therefore, either ‘Ma’ or ‘Va’ could be the code for ‘looking’.


95. C
Sol.

Father of my son’s father is Sabiya’s father-in-law. So, Sabiya’s father-in-law brother is Sabiya’s uncle.

Hence, option C is the correct answer.


96. B
Sol. From the information in the question,

Y is Great Grandmother of Z.
So, the correct answer is option B.
97. C
Sol. Woman’s father’s only daughter means the woman herself.
Therefore, the woman is the mother of that person.
www.gradeup.co

Hence, option C is the right answer. 


98. A
Sol. I. R = B > A > G (True)
II. F > E < N < K = B (False)
99. A
Sol. K≤P≤N≤B<T
100. A
Sol. H > J = K, K ≥ L, L > T, T < V: K ≥ L, L > T → K>T & H > J = K, K ≥ L→ H>L
Conclusions
I. K > T : True
II. L≤ H: Not True
101. D
Sol. 1) Less than two people watched the movie after B, who did not, watched the movie on Saturday.

2) Three people watched the movie between G and B.

3) E watched the movie before G.

4) Three people watched the movie between A and E.


www.gradeup.co

5) C watched the movie after A.

6) F watched the movie before D.

Therefore, A watched the movie on thursday.


102. B
Sol. 1) Less than two people watched the movie after B, who did not, watched the movie on Saturday.

2) Three people watched the movie between G and B.


www.gradeup.co

3) E watched the movie before G.

4) Three people watched the movie between A and E.

5) C watched the movie after A.

6) F watched the movie before D.

Therefore, option B is the correct answer.


103. D
Sol. 1) Less than two people watched the movie after B, who did not, watched the movie on Saturday.

2) Three people watched the movie between G and B.


www.gradeup.co

3) E watched the movie before G.

4) Three people watched the movie between A and E.

5) C watched the movie after A.

6) F watched the movie before D.

Therefore, option D is the correct answer.


104. D
www.gradeup.co

Sol. 1) Less than two people watched the movie after B, who did not, watched the movie on Saturday.

2) Three people watched the movie between G and B.

3) E watched the movie before G.

4) Three people watched the movie between A and E.

5) C watched the movie after A.

6) F watched the movie before D.


www.gradeup.co

Therefore, option D is the answer.


105. B
Sol. 1) Less than two people watched the movie after B, who did not, watched the movie on Saturday.

2) Three people watched the movie between G and B.

3) E watched the movie before G.

4) Three people watched the movie between A and E.


www.gradeup.co

5) C watched the movie after A.

6) F watched the movie before D.

Except option B, both people has one person in between.


Therefore, option B is the correct answer.
106. B
Sol. 1) C lives on an even-numbered floor above the fifth floor.

2) Two people live between D and C.

3) Three people live between G and D.


www.gradeup.co

4) Four people live between H and G.

5) A lives below H but not on an even-numbered floor.

6) E lives three floors below B.


www.gradeup.co

7) F lives above A.

(Here, case 2 will be eliminated)

Therefore, option B is the correct answer.


107. B
Sol. 1) C lives on an even-numbered floor above the fifth floor.
www.gradeup.co

2) Two people live between D and C.

3) Three people live between G and D.

4) Four people live between H and G.

5) A lives below H but not on an even-numbered floor.


www.gradeup.co

6) E lives three floors below B.

7) F lives above A.

(Here, case 2 will be eliminated)


www.gradeup.co

Therefore, option B is the correct answer.


108. A
Sol. 1) C lives on an even-numbered floor above the fifth floor.

2) Two people live between D and C.

3) Three people live between G and D.


www.gradeup.co

4) Four people live between H and G.

5) A lives below H but not on an even-numbered floor.

6) E lives three floors below B.


www.gradeup.co

7) F lives above A.

(Here, case 2 will be eliminated)

Therefore, option A is the correct answer.


109. D
Sol. 1) C lives on an even-numbered floor above the fifth floor.
www.gradeup.co

2) Two people live between D and C.

3) Three people live between G and D.

4) Four people live between H and G.

5) A lives below H but not on an even-numbered floor.


www.gradeup.co

6) E lives three floors below B.

7) F lives above A.

(Here, case 2 will be eliminated)


www.gradeup.co

Therefore, option D is the correct answer.


110. C
Sol. 1) C lives on an even-numbered floor above the fifth floor.

2) Two people live between D and C.

3) Three people live between G and D.


www.gradeup.co

4) Four people live between H and G.

5) A lives below H but not on an even-numbered floor.

6) E lives three floors below B.


www.gradeup.co

7) F lives above A.

(Here, case 2 will be eliminated)

Therefore, option C is the correct answer.


111. D
Sol. The first paragraph depicts how earlier, a consumer never bothered about the where and how of the commodities
he/she consumed. Similarly, the condition of the farmers was never a topic of discussion. As explained further in the
passage, this situation prevailed because the producers belonged to the colonies. We need to find a sentence from the
www.gradeup.co

passage (and not just the first paragraph) given as options, that would justify the ideas mentioned in the first
paragraph. Option D does this, thus, it is the correct answer.
112. A
Sol. Refer to the following lines of the passage:
1. "...in the late 1980s, you began to hear more about these farmers, encountering their stories on television or in
newspapers or even on the labels of the packages you bought." Television and newspapers are forms of digital and
print media respectively. Thus, statement I is correct.
2. "For centuries, trade propelled the colonial project, and exploitation was its very purpose." Thus, statement II is
also correct.
3. "Fairtrade was founded on the conviction that consumers could make the marketplace more moral." Thus,
statement III is incorrect.
Thus, option A is the correct answer.
113. C
Sol. Refer to the following lines from the passage:
1. "If companies pay farmers equitably, Fairtrade believes, other benefits cascade out as well." Thus, statement I is
correct.
2. "Producers must meet a number of standards to qualify for Fairtrade: rules about labour conditions, for
instance, or waste disposal." Thus, it cannot be said that farmers are not obliged to meet specified standards. 
3. "...for companies, the totality of their ethical responsibilities towards their producers is encapsulated by price.
Thus, statement III is correct.
Hence, option C is the correct answer.  
114. D
Sol. Refer to the following lines from the passage: 
1. "The reasons were manifold. Environmental awareness was on the rise." Thus, alternative 1 is correct.
2. "The prices of some commodities were crashing, placing agricultural incomes in even more acute peril than
usual." Thus, alternative 2 is correct.
3. There had already been small groups pushing for more equitable trade..." Hence, alternative 3 is incorrect.
Option E is the correct answer.
115. A
Sol. The error is in the first part of the sentence. "Trade" is a singular subject and requires a singular verb. Thus, "have"
cannot follow it. Also, the mention of "now" in the latter part of the sentence implies that the first part talks about the
past. Thus, the most appropriate verb would be "had".
www.gradeup.co

116. D
Sol. Amateur- non-professional
Insular- narrow-minded
Malodorous- having an unpleasant fragrance

The scent coming from a corpse would be nothing but unpleasant. Thus, option D is the correct answer.
117. A
Sol. The word ‘attack’ carries two meaning:
I. As a noun, it means “strong criticism of someone or something” (appropriately fills sentence I)

II. As a verb, it means “to try to hurt something or someone using force and violence” (appropriately fills sentence II)

III. The word ‘raid’ as a noun means “a surprise visit by police to arrest suspects or seize illicit goods” (appropriately
fills sentence III)

Thus, option A is the correct answer.


118. B
Sol. To resolve this, lets understand the context. Note that an outcry means a strong expression of public disapproval or
anger. Now in the given sentence, this resentment stands initiated/encouraged because of mercury dumping.

Now in the first blank we need a word that is synonymous to 'encourage'. Based on this we can shortlist option B
'incite' which means encourage or stir up and C 'provoke' which means stimulate or give rise to' . We know that an
outcry cannot be activated. Now based on this we can see that the second blank needs a verb in its base form to
maintain the parallel structure with 'affect' used in the sentence. Obviously 'determined' does not relate to it. Hence
option B is the correct answer.
The new sentence will be
The move to allow dumping of mercury provoked an outcry from residents of the area who fear that high levels of
mercury will affect their health and destroy ecologically sensitive forest area.
119. A
Sol. Agog- eager, excited
Erudite- learned
Attrition- the process of reducing something's strength or effectiveness through sustained attack or pressure.
Exclaimed- said
www.gradeup.co

Of all the given options, only "agog" fits here.

120. A
Sol. Statement (ii) is factually incorrect. The passage mentions IP to be the one that defines packet structures
that encapsulate the data to be delivered. Statement (iii) is also in contradiction with the passage. As per
the passage, when Vint Cerf and Bob Kahn introduced Transmission Control Program in 1974, IP was the
connectionless datagram service, while the sentence refers to TCP for being the connectionless datagram
service. Statement (i) is correct and encapsulates almost all the important roles played by Internet
Protocol. Therefore, option A is the correct answer.
121. A
Sol. Statement (ii) is completely out of context. As per the passage, the Great Depression was related to the
market crash and falling economy, and not with the literal depression. Further, the passage does not
specifies any group which suffered the most from the depression. Statement (iii) factually incorrect, as it
specifies the Great Depression to have occurred after the second world war, which is incorrect. Statement
(i) is correct and it encapsulates what the passage implies. As statement (i) is the only one which is
correct, therefore option A is the correct answer.
122. C
Sol. The summary of the paragraph must touch up these points:
• A black hole is very inefficient at converting its food to X-rays or any other form of radiation.
• A black hole has a stellar companion that sheds gas and forms a disk around the black hole.
• Those disks are very good at producing the X-rays detectable by powerful technology.
One may get confused between option B and option C so notice the word ‘augmenting’ in option B. Augmenting
means to make something greater by adding to it; increase. This is opposite of what a black hole does.
Therefore, option C is the apt answer.
123. D
Sol. Option A: The problem mentioned does not match to the problem mentioned in the paragraph.
Option B: It can’t be deciphered from the paragraph.
Option C: The paragraph does not say that ‘individuals who test positive for active tuberculosis (TB) but do not
initiate treatment.’
Option E: ‘India did have was a public health system’ can’t be deciphered from the paragraph.
From ‘actually take the prescribed course of medication….But the course of treatment is long: typically six months
www.gradeup.co

to a year…,’ one can say that option D is the most appropriate summary among the other options.
Therefore, option D is the apt answer.
124. D
Sol. ‘Harp on the same tune’ refers to ‘keep on repeating the same thing’.
125. B
Sol. ‘Gall and warmwood’ refers to something hateful which cause strong feelings of bitterness and
resentment.
126. C
Sol. ‘In the bad books’ - Having fallen out of favour with someone.
127. C
Sol. 'Fell off' makes the sentence grammatically correct as it means something falling out of something else.
128. D
Sol. The correction here is to omit the preposition 'of' from the boldened part; it is not required there and its presence
gives incoherent meaning to the statement.
129. B
Sol. In this sentence, a situation is being discussed in which the players go to other places to play matches for which they
have to travel. Roaming is for casual purposes so travel is a more suitable word here. Rest all the options have
subject verb disagreement.
130. B
Sol. The correct usage is “longer” as the sentence contains the word “than” representing a comparison between the time
taken to return a product to the online store and a local shop. Therefore, the correct answer is B.
131. C
Sol. The first sentence has to be D as it talks about the beginning of the competition. Option A is the next
statement at it connects with A and mentions the ending schedule, thus creating a timeline in which the
other statements will occur. B is obviously the last statement as it is the only one which is related to
statement 6. Thus the correct sequence is DACB.
132. B
Sol. D follows 1 as ‘the problem’ mentioned in D is stated in sentence 1. B is directly related to D as it explains
the problem in more details. Next in the sequence should be C as ‘the casual explanation’ is a continuity
of statement B. ‘Yet’ of A sums up to the author’s comments leading to 6.
Thus the correct order is DBCA.
133. A
www.gradeup.co

Sol. S has the name of the event, so it is clear that S is an introductory statement. P provides more information about it, so
SP is a pair. Q and R both talk about “causes”, so QR is also a pair. All these conditions are fulfilled in option A;
hence, option A is the correct answer.
134. B
Sol. Q introduces the term “SBM” and S gives us more information about it, so QS is a pair. R mentions a problem and
“this” in P refers to it so it can be concluded that RP is also a pair. All these conditions are fulfilled in option B;
hence, option B is the correct answer.
135. B
Sol. Option B has the incorrectly spelt word and its correct spelling is "auspices" which means with the protection or
support of someone or something, especially an organization. 

The meanings of the other words are:

Mississauga = Mississauga is a large Canadian city neighboring Toronto on Lake Ontario.

Torrent = a strong and fast-moving stream of water or other liquid. 

Amenable = Liable to answer to a higher authority. 


136. C
Sol. Option C has the correctly spelt word as "differentiate" which means mark as different.

The correct spellings of the other words along with their meanings are:-

Unprovable = not provable.

Superstitious = showing ignorance of the laws of nature and faith in magic or chance.

Jackpot = the cumulative amount involved in a game.


137. D
Sol. Option D has the incorrectly spelt word and its correct spelling is "subsequent" which means coming after something
in time; following.
www.gradeup.co

The meaning of other words are :-

Sophisticated = having or appealing to those having worldly knowledge and refinement and savoir-faire.

Specifically = in distinction from others.

Squeeze = use force to compress, making out of normal shape or condition.


138. D
Sol. Let's first learn the meaning of the given words:
Starkly = very obviously and clearly. 
e.g. An opinion survey less than a week after the debate starkly highlighted her effect on public thinking.
Obviously = unmistakably; visibly clear; in an evident manner.
e.g. The answer is obviously wrong.
Irreverently = showing a lack of respect for people or things that are generally taken seriously.
Innocently = in a way that means you are not guilty of a crime
Maturely = in a mature and responsible way
Hence, option D is the correct answer.
139. C
Sol. Compassionate(दयालु) = feeling or showing sympathy and concern for others.

Pathetic(निराश) = causing feelings of sadness, sympathy.

Aesthetic(सौंदर्यबोध) = concerned with beauty or the appreciation of beauty.

Sympathetic(सहानुभूतिपूर्ण) = feeling, showing or expressing sympathy.

Hence, option C is the correct answer.


140. D
Sol. Let's first learn the meanings of the words:

Horrify = to fill with horror; shock greatly.


e.g. "The very idea of this sort of thing would horrify the wimp establishment, both here and abroad"
www.gradeup.co

Affright = to frighten or scare someone.


 
Petrify = make (someone) so frightened that they are unable to move.

Appal = greatly dismay or horrify.

Soothe = to gently calm (a person or their feelings).


e.g. "It counted for something but did nothing to soothe her anger"

Hence, option D is the correct answer.


141. A
Sol. A man covers a total distance of 100 km on bicycle. For the first 2 hours, the speed was 20 km/hr and for the rest of
the journey, it came down to 10 km/hr.
In the first 2 hours, distance covered = 20 × 2 = 40 km
Remaining distance = 100 – 40 = 60km
Time required to cover this remaining distance = 60/10 = 6hr
Now total time taken to complete the total distance = 2 + 6 = 8hr

Average speed = Total distance/total time = 100/8 =


142. A
Sol. Let the usual speed  and distance

Time taken at usual speed

Time taken at 3/4th of usual speed  = 5/3 hours


Time taken at usual speed=  hours


www.gradeup.co

Time saved  hr  minutes

143. C
Sol. Let income of Monica
Expenditure
Saving
A.T.Q.
Her new income

New expenditure
Required,

144. C
Sol. Total work  units

Per day work of A = 24/8 = 3 units/day

Per day work of B = 24/12 = 2 units/day

Work done in two days  units

units of work will be completed in days


www.gradeup.co

Remaining work  = 4 units

B on the 9th day will do 2 units out of 4 units and remaining 2 units will be done by A in  day.

Total time  days


145. D
Sol. Let the days taken by P and Q to complete the whole work be p and q.

Therefore, according to the question;

 = 1 …(i) and

 = 1 …(ii) {as total work is taken as 1 here}

Since, (i) = (ii)

⇒ 4a + 18b = 6a + 12b {take 1/p as ‘a’ and 1/q as ‘b’}

⇒ a = 3b …(iii)

Put this value in eq. (i) or (ii)

∴ 4a + 18b = 1

⇒ 4 × 3b + 18b=1

⇒b=

⇒ q = 30 days

Similarly, a =
www.gradeup.co

⇒ p = 10 days

Hence, option D is the correct answer.

146. A
Sol. First discount = 20%
Price after first discount

= Rs.

= Rs.(1500 – 300) = Rs.1200


Let the additional discount be x%.

147. C
Sol.

 
 
 
148. B
Sol. A : B = (21 × 12) : (36 × x) = 1 : 1
⇒7:x=1:1
www.gradeup.co

⇒ x = 7 months

B was in the business for 7 months. He entered into business after 12 – 7 = 5 months.
149. C
Sol. Let P's present age  = 2x years

Q's present age = 3x years

According to the question,

10 years ago -

⇒ 10x−50 = 9x−30

⇒ x = 20

∴ P's present age  = 2×20 = 40 years

Q's present age = 3×20 = 60 years 

Option C is correct.
150. B
Sol. Let R% be the rate of simple interest then,

from question we can conclude that

⇒ 100R + 120R = 2200

⇒ 220R = 2200
www.gradeup.co

⇒ R = 10%

You might also like